You are on page 1of 83

Total Marks : 200

Online Prelims TEST - 28 (TEXTBOOK)


( InsightsIAS Mock Test Series for UPSC Preliminary Exam 2020 ) Mark Scored : 0

1 Consider the following statements.


1. Seventh Schedule of the Constitution states that ordinarily the use of the armed forces in the
maintenance of public order is outside the purview of the states.
2. While public order and police are under the state list, the state government may request the Union
government to make available armed forces to help restore public order.

Select the correct answer using the codes below.


A. 1 only
B. 2 only
C. Both 1 and 2
D. None of the above

Correct Answer : C

Answer Justification :

Justification and Learning: It is conceivable that a State Government is unable or unwilling to


suppress an internal disturbance and may even refuse to seek the aid of the armed forces of the
Union in the matter.

However, the Union Government, in view of its constitutional obligation, cannot be a silent
spectator when a finds the situation fast drifting towards anarchy or a physical breakdown of the
State administration. In such an unusual, yet not entirely an improbable event, the Union
Government may deploy its armed forces suo motu to deal with the disturbance and restore public
order.

The phrase "in aid of the civil power" in Entry 2A of List I and Entry 1 in List II signifies that the
deployment is in aid of the instrumentalities of the State charged with the maintenance of public
order. It does not necessarily imply that such deployment should take place only at the request of
the State Government.

While the Union Government has, under Article 355, all the powers that it may need to deal with an
internal disturbance, it cannot assume the sole responsibility for dealing with an internal
disturbance by superseding or excluding the State police and other authorities responsible for
maintaining public order. Neither can the Union Government deploy, in contravention of the wishes
of a State Government, its armed forces to deal with a relatively less serious public order problem
which is unlikely to escalate and which the State Government is confident of tackling.

It would not be constitutionally proper for the Union Government to take such measures except
when a national emergency under Article 352 or President's rule under Article 356 has been
proclaimed.

The use of the armed forces of the Union in the maintenance of public order (Entry I of
List II) has always been outside the purview of the States. Even before the insertion of Entry
2A in List I by the 42nd Amendment, the Union Government did have, by virtue of Entry 2 in List I,
exclusive control over its armed forces and had the power to deploy them in aid of the civil power
whether for maintaining public order or quelling an internal disturbance.

1
Total Marks : 200
Online Prelims TEST - 28 (TEXTBOOK)
( InsightsIAS Mock Test Series for UPSC Preliminary Exam 2020 ) Mark Scored : 0

While public order and police are under the state list, the state government may request the Union
government to make available armed forces to help restore public order.

Read more here http://interstatecouncil.nic.in/wp-content/uploads/2015/06/CHAPTERVII.pdf

Q Source:
https://www.insightsonindia.com/wp-content/uploads/2020/03/Insights-February-2020-Current-Affai
rs-Compilation.pdf

2 The report of the Fifteenth Finance Commission, along with an Action Taken Report, was recently
tabled in Parliament. Which of these criteria was used by the commission to arrive at the states’ share
in the divisible pool of taxes?
1. Population size
2. Tribal population
3. Size of state’s GDP
4. Industrialization factor

Select the correct answer using the codes below.


A. 1, 2 and 3 only
B. 1 only
C. 2, 3 and 4 only
D. 1, 2, 3 and 4

Correct Answer : B

Answer Justification :

Justification: The 15th FC has considered the 2011 population along with forest cover, tax effort,
area of the state, and “demographic performance” to arrive at the states’ share in the divisible pool
of taxes.

In order to reward population control efforts by states, the Commission developed a criterion for
demographic effort — which is essentially the ratio of the state’s population in 1971 to its fertility
rate in 2011 — with a weight of 12.5%.

The total area of states, area under forest cover, and “income distance” were also used by the FC to
arrive at the tax-sharing formula.

Learning: Key recommendations: The Commission has reduced the vertical devolution — the share
of tax revenues that the Centre shares with the states — from 42% to 41%. The Commission has
said that it intends to set up an expert group to initiate a non-lapsable fund for defence expenditure.

The population parameter used by the Commission has been criticised by the governments of the
southern states. • The previous FC used both the 1971 and the 2011 populations to calculate the
states’ shares, giving greater weight to the 1971 population (17.5%) as compared to the 2011
population (10%). • The use of 2011 population figures has resulted in states with larger
populations like UP and Bihar getting larger shares, while smaller states with lower fertility rates

2
Total Marks : 200
Online Prelims TEST - 28 (TEXTBOOK)
( InsightsIAS Mock Test Series for UPSC Preliminary Exam 2020 ) Mark Scored : 0

have lost out. • The combined population of the Bihar, Uttar Pradesh, Madhya Pradesh, Rajasthan
and Jharkhand is 47.8 crore.

Q Source:
https://www.insightsonindia.com/wp-content/uploads/2020/03/Insights-February-2020-Current-Affai
rs-Compilation.pdf

3 Consider the following statements.


1. Import of live animals falls is free of licensing requirements from the Director General of Foreign
Trade (DGFT).
2. Trade in livestock products is regulated as per the Foreign Trade Policy–Export Import Policy
(EXIM) of Government of India.

Select the correct answer using the codes below.


A. 1 only
B. 2 only
C. Both 1 and 2
D. None of the above

Correct Answer : B

Answer Justification :

Justification: The question comes in the backdrop of Coronavirus infections which started in a live
animal market in the Wuhan province of China.

Trade in livestock and livestock products are regulated as per the Foreign Trade Policy–Export
Import Policy (EXIM) of Government of India which is implemented by Department of Commerce.
However, to prevent ingress of Exotic Diseases through import of livestock and livestock products,
Department of Animal Husbandry, Dairying and Fisheries regulates trade in such products as per
provision of Section 3 and Section 3A of the Livestock Importation Act., 1898. Import of live
animal’s falls under the category of restricted list (it is not free to import) as per EXIM Policy for
which importer has to obtain license from Director General of Foreign Trade (DGFT).

The DGFT issues license on the recommendation of this Department. This Department makes a
decision on recommendation based on the risk analysis and the related germplasm policy. Central
Government is empowered to regulate, restrict and prohibit import of live animals in accordance
with Section 3 of the Live-stock Importation Act., 1898.

The department has notified the classes of animals that can be considered as “Live-stock” and
requirement of veterinary health certificate for their import and quarantine procedure of live
animals. Livestock products are categorized under Open General License (OGL) as per EXIM Policy.

Central Government is empowered to regulate, restrict and prohibit import of live-stock products in
accordance with Section 3A of the Live-stock Importation Act., 1898. I

Q Source: https://sip.nic.in/Files/Function%20of%20Trade%20Unit%202_new%20%201.pdf

3
Total Marks : 200
Online Prelims TEST - 28 (TEXTBOOK)
( InsightsIAS Mock Test Series for UPSC Preliminary Exam 2020 ) Mark Scored : 0

4 “Many Englishmen honestly consider themselves the trustees for India, and yet to what a condition
they have reduced our country!” was the view held by ‘X’ which made him reject the trusteeship thesis
of Gandhiji. He (‘X’) was

A. Sardar Vallabhbhai Patel


B. Pandit Nehru
C. Lal Gangadhar Tilak
D. Dadabhai Naoroji

Correct Answer : B

Answer Justification :

Justification: Gandhiji) had strongly disapproved of Jawaharlal's snap Independence


resolution passed in his absence at the Madras Congress (1927).

At Calcutta next year he was able to push through a compromise formula which accepted the
Nehru Report's dominion status objective provided the British granted it by the end of 1929,
failing which the Congress would be free to go in for Civil Disobedience and Purna Swaraj.

Gandhi tried to confine Congress activities during 1929 to constructive work in villages,
prohibition, and boycott of British goods, plus redress, along Bardoli lines, of 'specific
grievances'.

He encouraged public bonfires of foreign cloth (for which he was arrested in Calcutta in
March, and awarded a token fine), and toured the country collecting funds for khadi, but
repeatedly rejected pressures for any all-out struggle.

Jawaharlal graced the Lahore Congress (December 1929) with the first of his stirring
Presidential addresses, boldly sketching out a new internationalist and socially-radical
perspective for the freedom movement—a perspective so far confined to small Leftist sects.

He said - 'I must frankly confess that I am a socialist and a republican, and am no believer in
kings and princes, or in the order which produces the modern kings of industry”…..He
attacked Gandhi's pet 'trusteeship' solution for zamindar-peasant and capital-labour conflicts:

“Many Englishmen honestly consider themselves the trustees for India, and yet to
what a condition they have reduced our country!” was the view Nehru held which
made him reject the trusteeship thesis of Gandhiji.

Q Source: Based on past UPSC papers

4
Total Marks : 200
Online Prelims TEST - 28 (TEXTBOOK)
( InsightsIAS Mock Test Series for UPSC Preliminary Exam 2020 ) Mark Scored : 0

5 Observer status is a privilege granted by some organizations to non-members to give them an ability to
participate in the organization's activities. Consider the following about observers in the United
Nations (UN).
1. Observer status can be granted to an entity by a United Nations General Assembly resolution.
2. Observers have the right to speak at United Nations General Assembly meetings, but not to vote on
resolutions.

Select the correct answer using the codes below.


A. 1 only
B. 2 only
C. Both 1 and 2
D. None of the above

Correct Answer : C

Answer Justification :

Justification: Observer status is often granted by intergovernmental organizations (IGO) to non-


member parties and international nongovernmental organizations (INGO) that have an interest in
the IGO's activities. Observers generally have a limited ability to participate in the IGO, lacking the
ability to vote or propose resolutions.

The United Nations General Assembly may grant entities observer status. The United Nations
welcomes many international agencies, entities, and two non-member states as observers.
Observers have the right to speak at United Nations General Assembly meetings, but not to vote on
resolutions.

Non-member observer states are recognized as sovereign states, and are free to submit a petition to
join as a full member at their discretion. At present, the Holy See and Palestine are the only
observer states at the United Nations, although Switzerland also maintained such status until it
became a member state. Among others the Sovereign Military Order of Malta also has observer
status, although not as a state but as an entity.

Observer status is granted by a United Nations General Assembly resolution at some point in time.
Other international organizations (including other UN agencies) may also grant observer status.

Q Source: UN Website and related Wiki page

6 Consider the following statements.


Assertion (A): The location of the thermal equator is not identical to that of the geographic Equator.
Reason (R): The Northern and the Southern hemisphere receive vastly different levels and intensity
of solar insolation.

In the context of the above, which of these is correct?


A. A is correct, and R is an appropriate explanation of A.
B. A is correct, but R is not an appropriate explanation of A.
C. A is correct, but R is incorrect.

5
Total Marks : 200
Online Prelims TEST - 28 (TEXTBOOK)
( InsightsIAS Mock Test Series for UPSC Preliminary Exam 2020 ) Mark Scored : 0

D. Both A and R are incorrect.

Correct Answer : C

Answer Justification :

Justification: The location of the thermal equator is not identical to that of the geographic
Equator.

This is not due to the Level of insolation received in both hemispheres. They are virtually equal.

What creates the difference is the distribution of land and water that causes changes in the way
heat is distributed and transported across the planets.

Land dominated areas will tend to have a large variation in temperature and a higher average
temperature under the same conditions (as compared to a water dominated area).

Thus, the thermal equator lies a bit north of the geographical equator.

Q Source: Additional Research: 11th NCERT: Fundamentals of Physical Geography

7 Kaipeng and Malsom tribals clans, recently seen in news, belong to the state/UT of

A. Ladakh
B. Tripura
C. Sikkim
D. Uttarakhand

Correct Answer : B

Answer Justification :

Justification: The Tripura Tribal Areas Autonomous District Council (TTAADC) has passed
resolutions to codify the customary laws of three tribal clans- Mizo, Kaipeng and Malsom.

As per the Sixth Schedule, the four states viz. Assam, Meghalaya, Tripura and Mizoram contain the
Tribal Areas which are technically different from the Scheduled Areas. Though these areas fall
within the executive authority of the state, provision has been made for the creation of the District
Councils and regional councils for the exercise of the certain legislative and judicial powers.

More details are covered in a different question.

Q Source: In news

8 Consider the following statements.


1. During the Neolithic phase, the cultivation of plants and domestication of animals led to the

6
Total Marks : 200
Online Prelims TEST - 28 (TEXTBOOK)
( InsightsIAS Mock Test Series for UPSC Preliminary Exam 2020 ) Mark Scored : 0

emergence of village communities based on sedentary life.


2. The people of Neolithic Age used clothes made of cotton and wool.

Select the correct answer using the codes below.


A. 1 only
B. 2 only
C. Both 1 and 2
D. None of the above

Correct Answer : C

Answer Justification :

Justification: During the Neolithic phase, the cultivation of plants and domestication of animals led
to the emergence of village communities based on sedentary life.

There was a great improvement in technology of making tools and other equipments used by
man. Stone tools were now polished.

Mud brick houses were built instead of grass huts.

Large urns were used as coffins for the burial of the dead.

Wheat, barely, rice, millet were cultivated in different areas at different points of time. Rice
cultivation was extensive in eastern India.

The people of Neolithic Age used clothes made of cotton and wool

Domestication of animals, horticulture and primitive cultivation started during Mesolithic period
itself.

However, during Neolithic period, domestication of sheep, goats and cattle was widely prevalent.
Cattle were used for cultivation and for transport.

During Neolithic age, wheels were used to make pottery. Pottery was used for cooking as well as
storage of food grains.

Q Source: Page 13-14: TN 11th Standard History Textbook

9 The Union Cabinet has approved the creation of the 22nd Law Commission, which advises the
government on complex legal issues. Consider the following about it.
1. A retired Supreme Court judge or Chief Justice of a High Court will head the Commission.
2. Law and Legislative Secretaries in the Law Ministry will be the ex-officio members of the

7
Total Marks : 200
Online Prelims TEST - 28 (TEXTBOOK)
( InsightsIAS Mock Test Series for UPSC Preliminary Exam 2020 ) Mark Scored : 0

commission.
3. It has a term of five years.

Select the correct answer using the codes below.


A. 1 and 2 only
B. 2 and 3 only
C. 1, 2 and 3
D. 1 only

Correct Answer : A

Answer Justification :

Justification: It is an executive body established by an order of the Government of India.

Originally formed in 1955, the commission is reconstituted every three years and so far, 277 reports
have been submitted to the government.

Statement 1 and 2: Composition: Apart from having a full-time chairperson, the commission will
have four full-time members, including a member-secretary.

Law and Legislative Secretaries in the Law Ministry will be the ex-officio members of the
commission.

It will also have not more than five part-time members.

A retired Supreme Court judge or Chief Justice of a High Court will head the Commission.

Statement 3: With the cabinet approval, the law ministry will now notify the new panel, which will
have a term of three years.

Learning: Roles and functions:

1. The Law Commission shall, on a reference made to it by the Central Government or suo motu,
undertake research in law and review of existing laws in India for making reforms and enacting new
legislation.

2. It shall also undertake studies and research for bringing reforms in the justice delivery systems
for elimination of delay in procedures, speedy disposal of cases, reduction in cost of litigation, etc.

Q Source:
https://www.insightsonindia.com/wp-content/uploads/2020/03/Insights-February-2020-Current-Affai
rs-Compilation.pdf

10 Uniform Code of Pharmaceutical Marketing Practices (UCPMP)


1. is issued by the Medical Council of India
2. is a mandatory or binding code on pharmaceutical companies

8
Total Marks : 200
Online Prelims TEST - 28 (TEXTBOOK)
( InsightsIAS Mock Test Series for UPSC Preliminary Exam 2020 ) Mark Scored : 0

Select the correct answer using the codes below.


A. 1 only
B. 2 only
C. Both 1 and 2
D. None of the above

Correct Answer : D

Answer Justification :

Justification: It is a voluntary code issued by the Department Of Pharmaceuticals relating to


marketing practices for Indian Pharmaceutical Companies and as well medical devices industry.
Applicability: At present, the UCPMP Code is applicable on Pharmaceutical Companies, Medical
Representatives, Agents of Pharmaceutical Companies such as Distributors, Wholesalers, Retailers,
and Pharmaceutical Manufacturer's Associations.

Department of Pharmaceuticals (DoP) has yet again “requested companies to abide by Uniform
Code of Pharmaceutical Marketing Practices (UCPMP)”. Background: There have been several
instances of breach of the voluntary Uniform Code of Pharmaceutical Marketing Practices (UCPMP)
by pharma companies. There has also been the demand from the Indian Medical Association (IMA)
and doctors to make it mandatory.

Q Source: In news

11 With reference to the Asia/Pacific Group on Money Laundering (APG), of which India is a member,
consider the following statements.
1. It derives from the Financial Action Task Force (FATF) constitution treaty.
2. APG facilitates the enforcement of anti-money laundering and anti-terrorist financing standards set
out in the recommendations of the FATF.

Which of the above is/are correct?


A. 1 only
B. 2 only
C. Both 1 and 2
D. None

Correct Answer : B

Answer Justification :

Background: It was officially established as an autonomous regional anti-money laundering body


in February 1997 at the Fourth (and last) Asia/Pacific Money Laundering Symposium in Bangkok.

The APG's role includes assisting jurisdictions in the region to enact laws dealing with the proceeds
of crime, mutual legal assistance, confiscation, forfeiture and extradition.

9
Total Marks : 200
Online Prelims TEST - 28 (TEXTBOOK)
( InsightsIAS Mock Test Series for UPSC Preliminary Exam 2020 ) Mark Scored : 0

It also includes the provision of guidance in setting up systems for reporting and investigating
suspicious transactions and helping in the establishment of financial intelligence units.

Justification: S1: The APG is voluntary and autonomous. It does not derive from an international
treaty nor is it part of any international organization. India became a member of the APG in 1998.

S2: The purpose of the APG is to facilitate the adoption, implementation and enforcement of
internationally accepted anti-money laundering and anti-terrorist financing standards set out in the
recommendations of the FATF, which is a globally body focussed on money laundering.

The APG undertakes studies of methods and trends of money laundering and the financing of
terrorism in the Asia/Pacific region.

The APG allows for regional factors to be taken into account in the implementation of anti-money
laundering and anti-terrorist financing measures and provides for peer review by means of a mutual
evaluation process.

Q Source: http://dor.gov.in/preventionofmoneylaundering/apg-membership

12 This state has decided to impose a ban on the sale of compact fluorescent lamps (CFL) and
incandescent (filament) bulbs, and also a village – Peelikode, of this state became the first panchayat
in the country to be completely filament free. The state is

A. Andhra Pradesh
B. Maharashtra
C. Kerala
D. Telangana

Correct Answer : C

Answer Justification :

Justification: Kerala will impose a ban on the sale of compact fluorescent lamps (CFL) and
incandescent (filament) bulbs starting November this year as part of sustainable energy policy.

This is in line with the government project of ‘Filament-free Kerala’ envisaged in 2018 as part of the
state’s Urja Kerala mission.

What is filament-free Kerala project?

It will be implemented by the Kerala State Electricity Board (KSEB) and the Energy Management
Centre, Kerala. Consumers in the state can place orders for LED bulbs on the KSEB website in
exchange for existing filament bulbs.

Nine-watt LED bulbs are being sold at reduced prices by the government to encourage usage. Last
year, Peelikode in Kasaragod district became the first panchayat in the country to be completely
filamentfree.

10
Total Marks : 200
Online Prelims TEST - 28 (TEXTBOOK)
( InsightsIAS Mock Test Series for UPSC Preliminary Exam 2020 ) Mark Scored : 0

The project is also part of the long-term sustainable energy policy to reduce the dependence on
conventional energy sources and instead maximise potential on renewable sources like solar and
hydel power.

Learning: Key differences between LED and CFL: The major difference between the CFL and LED
is that in CFL the emission of light is because of the ionisation of mercury vapour. The mercury
vapour when ionise produces ultraviolet rays. These rays when collides with phosphorous coating
tube generates visible light.

Whereas in the LED it is because of the PN junction diode. When the forward current applies across
the diode, the recombination of the charge carrier takes place. This charge carrier gives energy in
the form of the heat and light.

Q Source:
https://www.insightsonindia.com/wp-content/uploads/2020/03/Insights-February-2020-Current-Affai
rs-Compilation.pdf

13 Consider the following statements.


1. Ripening is associated with change in composition of a fruit, i.e. conversion of sugar to starch.
2. During the ripening process, some fruits emit ethylene along with an increased rate of respiration.

Select the correct answer using the codes below.


A. 1 only
B. 2 only
C. Both 1 and 2
D. None of the above

Correct Answer : B

Answer Justification :

Justification: Ripening is the process by which fruits attain their desirable flavour, quality, colour,
palatable nature and other textural properties.

Ripening is associated with change in composition i.e. conversion of starch to sugar (not the
other way round).

On the basis of ripening behavior, fruits are classified as climacteric and non-climacteric fruits.

Climacteric: Climacteric fruits are defined as fruits that enter ‘climacteric phase’ after harvest
i.e. they continue to ripen. During the ripening process the fruits emit ethylene along with
increased rate of respiration.

Ripe fruits are soft and delicate and generally cannot withstand rigours of transport and
repeated handling. These fruits are harvested hard and green, but fully mature and are
ripened near consumption areas. Small dose of ethylene is used to induce ripening process

11
Total Marks : 200
Online Prelims TEST - 28 (TEXTBOOK)
( InsightsIAS Mock Test Series for UPSC Preliminary Exam 2020 ) Mark Scored : 0

under controlled conditions of temperature and humidity.

Q Source: www.agritech.tnau.ac.in

14 With reference to Chalukyan architecture, consider the following statements.


1. They built cave temples depicting both religious and secular themes.
2. No Chalukyan age mural paintings can be found.
3. The temples under the Chalukyas are an example of the Vesara style of architecture.

Select the correct answer using the codes below.


A. 1 only
B. 2 and 3 only
C. 1 and 3 only
D. 1, 2 and 3

Correct Answer : C

Answer Justification :

Justification: Chalukyas ruled large parts of southern and central India between the 6th and the
12th centuries.

They built cave temples depicting both religious and secular themes. The temples had beautiful
mural paintings also.

The temples under the Chalukyas are a good example of the Vesara style of architecture. This is
also called the Deccan style or Karnataka Dravida or Chalukyan style. It is a combination of Dravida
and Nagara styles.

Learning: During this period, they ruled as three related yet individual dynasties: 1. The earliest
dynasty, known as the "Badami Chalukyas", ruled from Vatapi (modern Badami) from the middle of
the 6th century.

After the death of Pulakeshin II, the Eastern Chalukyas became an independent kingdom in the
eastern Deccan. They ruled from Vengi until about the 11th century. 3. The Western Chalukyas, of
10th century ruled from Kalyani (modern Basavakalyan) until the end of the 12th century.

Q Source:
https://www.insightsonindia.com/wp-content/uploads/2019/11/Insights-October-2019-Current-Affair
s-Compilation.pdf

15 With respect to Guru Ravidas, consider the following statements.


1. He was a poet and saint associated with the Bhakti movement.
2. His devotional songs were included in the Sikh scriptures.
3. Mirabai was a disciple of Guru Ravidas.

12
Total Marks : 200
Online Prelims TEST - 28 (TEXTBOOK)
( InsightsIAS Mock Test Series for UPSC Preliminary Exam 2020 ) Mark Scored : 0

Select the correct answer using the codes below.


A. 1 only
B. 2 and 3 only
C. 1 and 3 only
D. 1, 2 and 3

Correct Answer : D

Answer Justification :

Justification: Facts about Guru Ravidas

1. He was a poet and saint of the Bhakti movement.

2. Born in Varanasi to the ‘untouchable’ leather tanners caste.

3. He is believed to be a disciple of the bhakti saint-poet Ramananda and a contemporary of the


bhakti saint-poet Kabir.

4. One of his famous disciples was the saint, Mirabai.

5. Ravidas' devotional songs were included in the Sikh scriptures, Guru Granth Sahib.

6. The Panch Vani text of the Dadupanthi tradition within Hinduism also includes numerous poems
of Ravidas.

7. Ravidas taught removal of social divisions of caste and gender, and promoted unity in the pursuit
of personal spiritual freedoms.

8. Among Ravidas’s moral and intellectual achievements were the conception of “Begampura”, a
city that knows no sorrow; and a society where caste and class have ceased to matter.

Q Source: 11th TN History Textbook

16 With respect to private member’s bills, consider the following statements.


1. A Private Member must give a notice before the Bill can be listed for introduction in the house.
2. While government Bills can be introduced and discussed on any day, private member’s Bills can be
introduced and discussed only on a certain day of the week.
3. No private member’s Bill has been passed by Parliament so far.

Select the correct answer using the codes below.


A. 1 and 2 only
B. 1, 2 and 3
C. 2 only
D. 3 only

Correct Answer : A

13
Total Marks : 200
Online Prelims TEST - 28 (TEXTBOOK)
( InsightsIAS Mock Test Series for UPSC Preliminary Exam 2020 ) Mark Scored : 0

Answer Justification :

Justification: Any MP who is not a Minister is referred to as a private member. Admissibility of a


private member’s Bill: The admissibility is decided by the Chairman for Rajya Sabha and Speaker in
the case of Lok Sabha.

The procedure is roughly the same for both Houses: • The Member must give at least a month’s
notice before the Bill can be listed for introduction. • The House secretariat examines it for
compliance with constitutional provisions and rules on legislation before listing.

While government Bills can be introduced and discussed on any day, private member’s Bills can be
introduced and discussed only on Fridays. Has a private member’s bill ever become a law? As per
PRS Legislative, no private member’s Bill has been passed by Parliament since 1970. To date,
Parliament has passed 14 such Bills, six of them in 1956. In the 14th Lok Sabha, of the over
300 private member’s Bills introduced, roughly four per cent were discussed, the remaining 96 per
cent lapsed without a single dialogue.

Q Source:
https://www.insightsonindia.com/wp-content/uploads/2020/03/Insights-February-2020-Current-Affai
rs-Compilation.pdf

17 Consider the following about Randomized Controlled Trial (RCT).


1. RCTs allow economists and other social science researchers to isolate the individual impact that a
certain factor alone has on the overall event.
2. RCTs can be used to test the effectiveness of various policy interventions to alleviate poverty.

Select the correct answer using the codes below.


A. 1 only
B. 2 only
C. Both 1 and 2
D. None of the above

Correct Answer : C

Answer Justification :

Justification: The new Economics Nobel laureates - Abhijit Banerjee, Esther Duflo and Michael
Kremer - are considered to be instrumental in using randomised controlled trials to test the
effectiveness of various policy interventions to alleviate poverty.

What is a randomised controlled trial?

It is an experiment that is designed to isolate the influence that a certain intervention or variable
has on an outcome or event.

Why is randomised controlled trial so popular?

14
Total Marks : 200
Online Prelims TEST - 28 (TEXTBOOK)
( InsightsIAS Mock Test Series for UPSC Preliminary Exam 2020 ) Mark Scored : 0

1. At any point in time, there are multiple factors that work in tandem to influence various social
events.

2. RCTs allow economists and other social science researchers to isolate the individual impact that
a certain factor alone has on the overall event.

3. For instance, to measure the impact that hiring more teachers can have on children’s learning,
researchers must control for the effect that other factors such as intelligence, nutrition, climate,
economic and social status etc., which may also influence learning outcomes to various degrees,
have on the final event.

4. Randomised controlled trials promise to overcome this problem through the use of randomly
picked samples. Significance: Many development economists believe that RCTs can help
governments to find, in a thoroughly scientific way, the most potent policy measures that could help
end poverty rapidly.

Q Source: In news

18 Arrange the following Asian geographical features from West to East.


1. Gobi Desert
2. Plateau of Tibet
3. Yellow Sea
4. Western Himalayas

Select the correct answer using the codes below.


A. 4213
B. 4231
C. 4123
D. 2143

Correct Answer : A

Answer Justification :

Justification:

15
Total Marks : 200
Online Prelims TEST - 28 (TEXTBOOK)
( InsightsIAS Mock Test Series for UPSC Preliminary Exam 2020 ) Mark Scored : 0

Q Source: Map-based questions: Asia

19 Consider the following statements.


1. Convertible Note can be used as an investment option by Indian start-ups that are looking for
raising funds apart from directors, relatives or shareholder’s of the Company.
2. A start-up company engaged in a sector where foreign investment requires Government approval
cannot issue convertible notes to any non-resident entity.
3. Start-ups cannot issue equity or debt instruments to Foreign Venture Capital Investors (FVCIs)
against receipt of foreign remittance.

Select the correct answer using the codes below.


A. 1 and 2 only
B. 1 only
C. 1, 2 and 3
D. None of the above

Correct Answer : B

Answer Justification :

Justification: Statement 1: Convertible Note is an instrument issued by a startup company


evidencing receipt of money initially as debt, which is repayable at the option of the holder, or
which is convertible into such number of equity shares of such startup company, within a period not
exceeding five years from the date of issue of the convertible note, upon occurrence of specified
events as per the other terms and conditions agreed to and indicated in the instrument.
16
Total Marks : 200
Online Prelims TEST - 28 (TEXTBOOK)
( InsightsIAS Mock Test Series for UPSC Preliminary Exam 2020 ) Mark Scored : 0

Convertible Note can be used as an investment option by Indian startups that are looking for raising
funds apart from directors, relatives or shareholder’s of the Company.

Convertible Note is a new instrument for availing credit from a Foreign national or Foreign Entity.
The condition and requirements of availing Convertible Note is detailed in FEMA Regulations.

Statement 2: A startup company engaged in a sector where foreign investment requires


Government approval may issue convertible notes to a non-resident only with approval of the
Government.

A startup company issuing convertible notes to a person resident outside India shall receive the
amount of consideration by inward remittance through banking channels or by debit to the NRE /
FCNR (B) / Escrow account maintained by the person concerned in accordance with the Foreign
Exchange Management (Deposit) Regulations, 2016, as amended from time to time

Statement 3: A Foreign Venture Capital Investor (FVCI) is an investor incorporated or established


outside India who can invest either in a Domestic Venture Capital Fund or a Venture
Capital Undertaking (Domestic Unlisted Company). Foreign equity player or Foreign Venture can
also invest in India directly under FDI Scheme.

Start-ups can issue equity or equity linked instruments or debt instruments to FVCI against receipt
of foreign remittance, as per the FEMA Regulation. In addition, startups can issue convertible notes
to person resident outside India subject to the following conditions:

Q Source: https://mofpi.nic.in/sites/default/files/cfpc_2017_final_released_28.8.17.pdf

20 Consider the following statements. BASIC group countries put together account for about
1. One-third of the World’s geographical area (land based)
2. Two-third of World’s human population

Select the correct answer using the codes below.


A. 1 only
B. 2 only
C. Both 1 and 2
D. None of the above

Correct Answer : A

Answer Justification :

Justification and Learning: The BASIC group was formed as the result of an agreement signed by
the four countries on November 28, 2009. They are a bloc of four large newly industrialized
countries – Brazil, South Africa, India and China.

Significance of the grouping:

Objectives: The signatory nations have a broadly common position on reducing greenhouse gas
emissions and raising the massive funds that are needed to fight climate change.
17
Total Marks : 200
Online Prelims TEST - 28 (TEXTBOOK)
( InsightsIAS Mock Test Series for UPSC Preliminary Exam 2020 ) Mark Scored : 0

The BASIC countries constituted one of the parties in the Copenhagen Accord reached with the US-
led grouping; the Accord, was, however, not legally binding.

The BASIC group wields considerable heft purely because of the size of the economies and
populations of the member countries.

Brazil, South Africa, India and China put together has one-third of the world’s geographical area
and nearly 40% of the world’s population, and when they unitedly speak in one voice this shows
their determination.

BASIC is one of several groups of nations working together to fight climate change and carry out
negotiations within the UNFCCC.

Q Source: Important international organizations

21 ‘The Gandhian Challenge’, launched by NITI Aayog is a platform for promoting

A. Hygiene and sanitation in schools


B. Innovative solutions for a sustainable India
C. Policy coordination amongst local bodies
D. Efficient administration at a cluster level within a particular state

Correct Answer : B

Answer Justification :

Justification: On the 150th birth Anniversary of Mahatma Gandhi, AIM, NITI Aayog’s Atal
Tinkering Labs (ATL) and UNICEF India, including Generation Unlimited, have launched ‘The
Gandhian Challenge’.

What is it?

This innovation challenge provides a platform for every child across India to ideate innovative
solutions for a sustainable India of their dreams, using Gandhi’s principles.

The winners of The Gandhian Challenge will be awarded in New Delhi by NITI Aayog’s Atal
Innovation Mission and UNICEF on the occasion of Children’s Day in November.

The contest – open for every child in India from 2 October to 20 October – also celebrates 70 years
of partnership between Government of India and UNICEF India to enable Every Right for Every
Child.

Ideas and solutions to the Gandhian Challenge may be expressed through broad categories: Art &
Innovation (Letters, poems, painting, videos and photos, among others) and Science, Technology &
Innovation (Robotics, IoT, sensors and 3D printers, among others).

Q Source:
https://www.insightsonindia.com/wp-content/uploads/2019/11/Insights-October-2019-Current-Affair

18
Total Marks : 200
Online Prelims TEST - 28 (TEXTBOOK)
( InsightsIAS Mock Test Series for UPSC Preliminary Exam 2020 ) Mark Scored : 0

s-Compilation.pdf

22 A Geotail would not be formed if there was no


1. Solar wind
2. Magnetic field produced by Earth

Select the correct answer using the codes below.


A. 1 only
B. 2 only
C. Both 1 and 2
D. None of the above

Correct Answer : C

Answer Justification :

Justification: An instrument on Chandrayaan-2, CLASS, designed to detect signatures of elements


in the Moon’s soil, had detected charged particles during the mission. This happened in September,
during the orbiter’s passage through the “geotail”.

A Geotail is a region in space that allows the best observations. The region exists as a result of the
interactions between the Sun and Earth.

How is it formed?

1. The Sun emits the solar wind, which is a continuous stream of charged particles.

2. These particles are embedded in the extended magnetic field of the Sun. Since the Earth has a
magnetic field, it obstructs the solar wind plasma.

3. This interaction results in the formation of a magnetic envelope around Earth.

4. On the Earth side facing the Sun, the envelope is compressed into a region that is approximately
three to four times the Earth radius.

5. On the opposite side, the envelope is stretched into a long tail, which extends beyond the orbit of
the Moon.

6. It is this tail that is called the geotail.

7. Once every 29 days, the Moon traverses the geotail for about six days.

Q Source:
https://www.insightsonindia.com/wp-content/uploads/2019/11/Insights-October-2019-Current-Affair
s-Compilation.pdf

23 Lunar Reconnaissance Orbiter (LRO) is a mission of

19
Total Marks : 200
Online Prelims TEST - 28 (TEXTBOOK)
( InsightsIAS Mock Test Series for UPSC Preliminary Exam 2020 ) Mark Scored : 0

A. ISRO
B. Space X
C. NASA
D. European Space Agency

Correct Answer : C

Answer Justification :

Justification: It is a NASA mission to the moon within the Lunar Precursor and Robotic Program
(LPRP) in preparation for future manned missions to the moon and beyond (Mars). LRO is the first
mission of NASA’s `New Vision for Space Exploration’.

The objectives of LRO are to: • Identify potential lunar resources. • Gather detailed maps of the
lunar surface. • Collect data on the moon’s radiation levels. • Study the moons polar regions for
resources that could be used in future manned missions or robotic sample return missions. •
Provide measurements to characterize future robotic explorers, human lunar landing sites and to
derive measurements that can be used directly in support of future Lunar Human Exploration
Systems.

Q Source: NASA’s website

24 Consider the following statements about the provisions of Article 13 and related judgments of the
Supreme court with regards to its implementation.
1. Statutory instruments in the nature of delegated legislation come under the ambit of Article 13.
2. A constitutional amendment is not to be recognized as a law and thus cannot be challenged under
this Article.
3. Non-legislative sources of law, that is, custom or usage having the force of law can be challenged
under Article 13.

Select the correct answer using the codes below.


A. 1 and 3 only
B. 1 only
C. 2 and 3 only
D. 2 only

Correct Answer : A

Answer Justification :

Justification: Article 13 declares that all laws that are inconsistent with or in derogation of any of
the fundamental rights shall be void. In other words, it expressively provides for the doctrine of
judicial review.

This power has been conferred on the Supreme Court (Article 32) and the high courts (Article 226)
that can declare a law unconstitutional and invalid on the ground of contravention of any of the

20
Total Marks : 200
Online Prelims TEST - 28 (TEXTBOOK)
( InsightsIAS Mock Test Series for UPSC Preliminary Exam 2020 ) Mark Scored : 0

Fundamental Rights.

The term ‘law’ in Article 13 has been given a wide connotation so as to include the following:

(a) Permanent laws enacted by the Parliament or the state legislatures;

(b) Temporary laws like ordinances issued by the president or the state governors;

(c) Statutory instruments in the nature of delegated legislation (executive legislation) like
order, bye-law, rule, regulation or notification; and

(d) Non-legislative sources of law, that is, custom or usage having the force of law.

Thus, not only a legislation but any of the above can be challenged in the courts as violating a
Fundamental Right and hence, can be declared as void.

Further, Article 13 declares that a constitutional amendment is not a law and hence cannot be
challenged. However, the Supreme Court held in the Kesavananda Bharati case2 (1973) that a
Constitutional amendment can be challenged on the ground that it violates a fundamental
right that forms a part of the ‘basic structure’ of the Constitution and hence, can be declared
as void.

Q Source: Ch 7: IPML

25 National Policy on Universal Electronic Accessibility is based on India’s commitments to and has a
bearing on
1. The United Nations Convention for the Rights of Persons with Disabilities
2. The Proclamation on the Full Participation and Equality of People with Disabilities in the Asia
Pacific Region, 1993
3. The Biwako Millennium Framework for action towards an inclusive, barrier free and rights based
society, 2002
4. Beijing Declaration and Platform for Action (PFA) (1995)

Select the correct answer using the codes below.


A. 1 and 2 only
B. 3 and 4 only
C. 1, 2 and 3 only
D. 1, 2, 3 and 4

Correct Answer : C

Answer Justification :

Justification: Statement 4: Adopted by governments at the 1995 Fourth World Conference on


Women, this document sets forth governments’ commitments to enhance women’s rights.

Statement 1, 2 and 3: The National Policy ("Policy") on Electronic Accessibility recognizes the need
to eliminate discrimination on the basis of disabilities and to facilitate equal access to Electronics &

21
Total Marks : 200
Online Prelims TEST - 28 (TEXTBOOK)
( InsightsIAS Mock Test Series for UPSC Preliminary Exam 2020 ) Mark Scored : 0

ICTs. The National Policy also recognizes the diversity of differently abled persons and provides for
their specific needs. The Policy covers accessibility requirements in the area of Electronics & ICT. It
recognizes the need for ensuring that accessibility standards and guidelines and universal design
concepts are adopted and adhered to.

The Policy anchors to: (i) The Constitution of India. (ii) The United Nations Convention for the
Rights of Persons with Disabilities ratified by India on October 01, 2007 (UNCRP D - Article 9 and
others). (iii) Persons with Disabilities (Equal Opportunities, Protection of Rights and Full
Participation) Act, 1995. (iv) The Proclamation on the Full Participation and Equality of People with
Disabilities in the Asia Pacific Region, 1993. (v) The Biwako Millennium Framework for action
towards an inclusive, barrier free and rights based society, 2002. (vi) National Policy for Persons
with Disabilities of the Government of India (2006).

Learning: The following strategies are envisaged for the implementation of the Policy: (i) Creating
awareness on universal electronics accessibility & universal design (ii) Capacity building and
infrastructure development (iii) Setting up of model Electronics & ICTs Centres for providing
training and demonstration to special educators and physically as well as mentally challenged
persons (iv) Conducting Research & Development, use of innovation, ideas, technology etc. whether
indigenous or outsourced from abroad (v) Developing programmes and schemes with greater
emphasis for differently abled women/children (vi) Developing procurement guidelines for
Electronics & ICTs for Accessibility and Assistive needs.

Q Source:
https://meity.gov.in/writereaddata/files/National%20Policy%20on%20Universal%20Electronics%281
%29_0.pdf

26 Gautama Buddha attained Parinirvana in

A. Piprahwa
B. Kushinagara
C. Sanchi
D. Vaisali

Correct Answer : B

Answer Justification :

Justification: Statement 1: Piprahwa is a village near Birdpur in Siddharthnagar district of Uttar


Pradesh. Piprahwa and its surroundings are of great significance to the Buddhist religion. It is one
of the sites that is linked directly to the early life of the Buddha as it is thought to be the location of
the ancient city of Kapilavastu.

According to Pali texts and ancient Buddhist traditions Piprahwa is also the site of one of eight
stupas constructed over the remains of Lord Buddha. Piprahwa is to be found about nine kilometres
north of Birdpur close to the border that separates India from Nepal.

Statement 2: Known as Kushavati in the ancient times, Kushinagar is where Gautama Buddha

22
Total Marks : 200
Online Prelims TEST - 28 (TEXTBOOK)
( InsightsIAS Mock Test Series for UPSC Preliminary Exam 2020 ) Mark Scored : 0

attained Parinirvana, which occurs upon death of a body of someone who has attained nirvana
during their lifetime. Many of the ruins of stupas here are dated as early as 3rd and 5th century
B.C.

Statement 4: The ancient city of Vaisali, which is now an archeological site, is where Gautama
Buddha preached his last sermon before his death in 483 B.C. The second Buddhist council was also
held here in 383 B.C. and the city contains one of the best preserved Pillars of Ashoka, topped by a
single Asiatic lion.

Q Source: Major Buddhist sites

27 The objectives of Rashtriya Gokul Mission (RGM) include


1. Development and conservation of indigenous breeds
2. Distribution of disease free high genetic merit bulls for natural service
3. To arrange quality Artificial Insemination (AI) services at farmers’ doorstep
4. To create e-market portal for bovine germplasm for connecting breeders and farmers

Select the correct answer using the codes below.


A. 1, 2 and 3 only
B. 1, 2 and 4 only
C. 3 and 4 only
D. 1, 2, 3 and 4

Correct Answer : D

Answer Justification :

Justification: Rashtriya Gokul Mission (RGM) has been launched in December 2014 with an outlay
of Rs 2025 crore for development and conservation of indigenous breeds through selective breeding
in the breeding tract and genetic upgradation of nondescript bovine population. The scheme
comprises of two components namely National Programme for Bovine Breeding (NPBB) and
National Mission on Bovine Productivity (NMBP).

India has 299.6 million bovine population out of which 190.9 million are cattle and 108.7 million are
buffaloes. Of this, 80% of cattle are indigenous and Non- descript breeds. The bovine genetic
resource of India is represented by 41 registered indigenous breeds of cattle and 13 registered
buffalo breeds. Indigenous bovine are robust and resilient and are particularly suited to the climate
and environment of their respective breeding tracts. The milk of indigenous animals is high in fat
and SNF content.

Objectives of RGM : ➢ Development and conservation of indigenous breeds; ➢ Breed improvement


programme for indigenous breeds so as to improve the genetic makeup and increase the stock; ➢
Enhancing milk production and productivity of bovine population by increasing disease free high
genetic merit female population and check on spread of diseases ; ➢ Upgrading nondescript cattle
using elite indigenous breeds like Gir, Sahiwal, Rathi, Deoni, Tharparkar, Red Sindhi ;

➢ Distribution of disease free high genetic merit bulls for natural service; ➢ To bring all breedable

23
Total Marks : 200
Online Prelims TEST - 28 (TEXTBOOK)
( InsightsIAS Mock Test Series for UPSC Preliminary Exam 2020 ) Mark Scored : 0

females under organised breeding through AI or natural service using germ plasm of high genetic
merits; ➢ To arrange quality Artificial Insemination (AI) services at farmers’ doorstep; ➢ To create e-
market portal for bovine germplasm for connecting breeders and farmers; ➢ To increase trade of
livestock and livestock products by meeting out sanitary and phyto sanitary (SPS) issues; ➢ To select
breeding bulls of high genetic merit at a young age through application of genomics.

Q Source: https://www.nddb.coop/ndpi/about/brief

28 Recent discovery of new moons of this planet makes it the planet with the highest number of moons
in the Solar system:

A. Jupiter
B. Neptune
C. Saturn
D. Uranus

Correct Answer : C

Answer Justification :

Justification: Recently the discovery of 20 new moons of Saturn has made Saturn the planet with
the highest number of moons (82). The previous record- holder, Jupiter, has 79.

This was announced by the International Astronomical Union’s Minor Planet Center.

Of Saturn’s 20 newfound moons, 17 are retrograde, meaning they orbit in the opposite direction
that Saturn rotates. Three orbit in the same direction that Saturn spins. Two of those prograde
moons orbit fairly close to the planet while one oddball is farther out

Q Source: In news

29 C40 cities World Mayors Summit was held in Copenhagen sometime ago. It was related primarily to
the issue of

A. Immigration
B. Climate Change
C. Unemployment
D. Energy efficiency

Correct Answer : B

Answer Justification :

Justification: C40 is a network of the world’s megacities committed to addressing climate change.
The C40 Cities Climate Leadership Group (C40) is a group of 94 cities around the world.

24
Total Marks : 200
Online Prelims TEST - 28 (TEXTBOOK)
( InsightsIAS Mock Test Series for UPSC Preliminary Exam 2020 ) Mark Scored : 0

C40 is focused on tackling climate change and driving urban action that reduces greenhouse gas
emissions and climate risks, while increasing the health, wellbeing and economic opportunities of
urban citizens.

Composition: While C40 originally targeted megacities for their greater capacity to address climate
change, C40 now offers three types of membership categories to reflect the diversity of cities taking
action to address climate change, i.e, Megacities, Innovator cities and Observer cities.

Criteria: The categories consider such characteristics as population size, economic output,
environmental leadership, and the length of a city’s membership.

Q Source: In news

30 The saint “Birbhan” is associated with which of these movements/cults of Medieval India?

A. Nathpanthis
B. Satnami movement
C. Dharmadasi cult
D. Firdausia Cult

Correct Answer : B

Answer Justification :

Justification: Satnamis originally were a militant sect of Hindu Worshippers.

Founded by a saint named “Birbhan” in 1657 in Narnaul in Haryana.

The major religious activity of this sect is to chant and meditate the true names (Sat-Nam) of
God, specially Rama and Krishna.

This sect is thought to be an offshoot of Ravidasi sect and comprised of lower strata of the
Hindu society, particularly, leather workers, sweepers, carpenters, Goldsmiths etc.

The followers of this sect kept their heads shaven (thus called Mundiyas) and abstained from
liquor and meat.

The religious granth of the Satnamis is called Pothi.

Q Source: Important religious movements: Medieval India

31 Consider the following statements.

25
Total Marks : 200
Online Prelims TEST - 28 (TEXTBOOK)
( InsightsIAS Mock Test Series for UPSC Preliminary Exam 2020 ) Mark Scored : 0

1. The Sarayu river originates from Lake Baikal in the Himalayas.


2. Sarayu finds mentions in the Vedas.
3. Ayodhya is situated on the banks of Sarayu river.

Select the correct answer using the codes below.


A. 1, 2 and 3
B. 2 and 3 only
C. 1 and 3 only
D. 1 and 2 only

Correct Answer : B

Answer Justification :

Justification: Sarayu flows through Uttarakhand and Uttar Pradesh. This river is of ancient
significance, finding mentions in the Vedas and the Ramayana. The Sarayu originates from Lake
Mansarovar in the Himalayas and is also known as the Ghaghra and the Manas Nandini. It merges
with the Ganga in Bihar’s Saran district.

It forms at the confluence of the Karnali and Mahakali in Bahraich District. Ayodhya is situated on
the banks of this river. It flows through the Kumaon himalayas.

5.50 lakh earthen lamps on the banks of Saryu River has helped Uttar Pradesh enter into the
Guinness Books of World Records for lightening of record number of diyas on any occasion.

Q Source: In news

32 B N Srikrishna committee constituted by the Ministry of Electronics and Information Technology


(MeitY), Government of India, that submitted its report sometime back, concerned the issue of

A. Data protection and data privacy


B. Innovation in Information and Communication Technology (ICT)
C. International coordination in ICT development
D. Generating skilled manpower in ICT

Correct Answer : A

Answer Justification :

Justification: On Juy 31, 2017, the Government of India appointed a 10-member Committee of
Experts headed by Justice B.N. Srikrishna, former Judge of the Supreme Court of India "to identify
key data protection issues in India and recommend methods of addressing them".

The terms of reference of the Committee were to "to study various issues relating to data protection
in India", and "to make specific suggestions for consideration of the Central Government on
principles to be considered for data protection in India and suggest a draft data protection bill".

26
Total Marks : 200
Online Prelims TEST - 28 (TEXTBOOK)
( InsightsIAS Mock Test Series for UPSC Preliminary Exam 2020 ) Mark Scored : 0

Learning: Here are the major highlights of the report

The law will have jurisdiction over the processing of personal data if such data has been used,
shared, disclosed, collected or otherwise processed in India.

Additionally, personal data collected, used, shared, disclosed or otherwise processed by


companies incorporated under Indian law will be covered, irrespective of where it is actually
processed in India. However, the data protection law may empower the Central Government
to exempt such companies which only process the personal data of foreign nationals not
present in India.

The law will not have retrospective application and it will come into force in a structured and
phased manner. The Aadhaar Act needs to be amended to bolster data protection.

The data protection law will set up a DPA which will be an independent regulatory body
responsible for the enforcement and effective implementation of the law. The Central
Government shall establish an appellate tribunal or grant powers to an existing appellate
tribunal to hear and dispose of any appeal against an order of the DPA.

Penalties may be imposed for violations of the data protection law. The penalties imposed
would be an amount up to the fixed upper limit or a percentage of the total worldwide
turnover of the preceding financial year, whichever is higher.

The state can process data without consent of the user on ground of public welfare, law and
order, emergency situations where the individual is incapable of providing consent,
employment, and Reasonable purpose.

The law will cover processing of personal data by both public and private entities.

Sensitive personal data will include passwords, financial data, health data, official identifier,
sex life, sexual orientation, biometric and genetic data, and data that reveals transgender
status, intersex status, caste, tribe, religious or political beliefs or affiliations of an individual.
However, the DPA will be given the residuary power to notify further categories in
accordance with the criteria set by law.

Consent will be a lawful basis for processing of personal data. However, the law will adopt a
modified consent framework which will apply a product liability regime to consent thereby
making the data fiduciary liable for harms caused to the data principal.

Cross border data transfers of personal data, other than critical personal data, will be through
model contract clauses containing key obligations with the transferor being liable for harms

27
Total Marks : 200
Online Prelims TEST - 28 (TEXTBOOK)
( InsightsIAS Mock Test Series for UPSC Preliminary Exam 2020 ) Mark Scored : 0

caused to the principal due to any violations committed by the transferee. Personal data
determined to be critical will be subject to the requirement to process only in India (there will
be a prohibition against cross border transfer for such data).

Q Source: https://meity.gov.in/writereaddata/files/Press_Brief_Data_Protection_1Aug17.pdf

https://economictimes.indiatimes.com/news/politics-and-nation/justice-bn-srikrishna-committee-sub
mits-report-on-data-protection-herere-the-highlights/articleshow/65164663.cms?from=mdr

33 The mission outlined in National Policy for Software Products (NPSP), 2019, includes
1. Enhancing the share of ICT to 20% of India’s GDP by 2022
2. To nurture several technology start-ups in software product industry including in Tier-II and Tier-III
towns
3. Developing sectoral and strategically located software product development clusters having
integrated ICT infrastructure

Select the correct answer using the codes below.


A. 1 and 2 only
B. 2 and 3 only
C. 1, 2 and 3
D. 3 only

Correct Answer : B

Answer Justification :

Background: The Software product ecosystem is characterized by innovations, Intellectual


Property (IP) creation and large value addition increase in productivity, which has the potential to
significantly boost revenues and exports in the sector, create substantive employment and
entrepreneurial opportunities in emerging technologies and leverage opportunities available under
the Digital India Programme, thus, leading to a boost in inclusive and sustainable growth.

According to NASSCOM the sector at present is generating an estimated revenue of around USD
168 billion, including export of USD 126 billion on an annual basis, which is around 8% contribution
to India’s GDP. The industry is also one of the largest organised sector employers, generating
nearly 14 million direct and indirect jobs. It is further estimated that the industry can contribute up
to 350 billion USD accounting to ~10 percent of India’s GDP by 2025.

Justification: The Policy will lead to the formulation of several schemes, initiatives, projects and
measures for the development of Software products sector in the country as per the roadmap
envisaged therein.

To achieve the vision of NPSP-2019, the Policy has the following five Missions:

1. To promote the creation of a sustainable Indian software product industry, driven by


intellectual property (IP), leading to a ten-fold increase in India share of the Global Software
28
Total Marks : 200
Online Prelims TEST - 28 (TEXTBOOK)
( InsightsIAS Mock Test Series for UPSC Preliminary Exam 2020 ) Mark Scored : 0

product market by 2025.

2. To nurture 10,000 technology startups in software product industry, including 1000 such
technology startups in Tier-II and Tier-III towns & cities and generating direct and in-direct
employment for 3.5 million people by 2025.

3. To create a talent pool for software product industry through (i) up-skilling of 1,000,000 IT
professionals, (ii) motivating 100,000 school and college students and (iii) generating 10,000
specialized professionals that can provide leadership.

IV. To build a cluster-based innovation driven ecosystem by developing 20 sectoral and strategically
located software product development clusters having integrated ICT infrastructure, marketing,
incubation, R&D/testbeds and mentoring support.

5. In order to evolve and monitor scheme & programmes for the implementation of this policy,
National Software Products Mission will be set up with participation from Government,
Academia and Industry.

Q Source: https://meity.gov.in/writereaddata/files/national_policy_on_software_products-2019.pdf

https://pib.gov.in/Pressreleaseshare.aspx?PRID=1566747

34 Consider the following statements. Ecomark is a certification mark issued


1. by the Bureau of Indian Standards (BIS)
2. for both electronic products and appliances

Select the correct answer using the codes below.


A. 1 only
B. 2 only
C. Both 1 and 2
D. None of the above

Correct Answer : C

Answer Justification :

Justification: Ecomark is issued by the BIS to products (not just appliances) conforming to a set of
standards aimed at the least impact on the ecosystem.

One of the purposes of the mark is increasing awareness among the consumers towards reducing
environment impact.

A wide variety of criteria are taken into account: Production process including source of raw

29
Total Marks : 200
Online Prelims TEST - 28 (TEXTBOOK)
( InsightsIAS Mock Test Series for UPSC Preliminary Exam 2020 ) Mark Scored : 0

material; Case of Natural Resources; Likely impact on the environment; Energy conservation in the
production of the product etc.

Learning: For the implementation of the scheme, BIS is responsible for the following functions:

i) Assessment of the product for ECO Mark, certification of the product for award of ECO Mark.

ii) Renewal, suspension and cancellation of the licence.

iii) Products certified as eligible for the ECO Mark shall also carry the ISI Mark (except for leather)
for quality, safety and performance of the product and shall be licensed to carry the ECO Mark for a
prescribed time period after which it shall be reassessed.

iv) Undertaking inspections and taking samples for analysis of any material or substance in relation
to which the BIS – ECO Mark has been used as may be necessary for proper implementation of ECO
Mark. For this purpose the Standard Mark of Bureau would be a single mark having a combination
of the ISI Mark and the ECO Logo

Q Source: http://envfor.nic.in/legis/others/ecomark.html

https://bis.gov.in/index.php/product-certification/operation-of-eco-mark-scheme/

35 Consider the following statements. The government has mandated giving procurement preference to
certain locally produced cyber security products
1. in some cases of public procurement
2. where intellectual property rights are owned by companies or start-ups incorporated in India

Select the correct answer using the codes below.


A. 1 only
B. 2 only
C. Both 1 and 2
D. None of the above

Correct Answer : C

Answer Justification :

Justification: The government has mandated giving preference to locally produced cyber security
products in all public procurement where intellectual property rights are owned by companies or
start-ups incorporated in India.

The notification is based on Public Procurement (Preference to Make in India) Order 2017 which
aims at enhancing income and employment in the country.

According to the order, the preference will be granted to a company which is incorporated and
registered in India or startup firms that meet the definition as prescribed by the Department of
Industrial Policy and Promotion (DIPP), provided the revenue from the product and intellectual
property (IP) licensing accrues to the firm in India.

30
Total Marks : 200
Online Prelims TEST - 28 (TEXTBOOK)
( InsightsIAS Mock Test Series for UPSC Preliminary Exam 2020 ) Mark Scored : 0

Firms claiming to own IP for the product will be required to furnish evidence of the ownership as
per the rule.

However, the order notes that IP registration is not mandatory in the country but a firm claiming
benefit under the order should have the right to use and commercialise product without third party
consents, distribute and modify it.

Q Source:
https://meity.gov.in/public-procurement-preference-make-india-order-2019-cyber-security-products

https://economictimes.indiatimes.com/small-biz/startups/newsbuzz/govt-to-give-preference-to-cyber-
security-products-by-domestic-firms/articleshow/64996614.cms?from=mdr

36 The vast majority of the highest mountain peaks in the World are located near

A. Ural region, Eurasia


B. Nepal and Tibet
C. Central Antarctica
D. Andes chain, South America

Correct Answer : B

Answer Justification :

Learning: There are at least 109 mountains on Earth with elevations greater than 7,200 metres
(23,622 ft) above sea level. The vast majority of these mountains are located on the edge of the
Nepal and Tibet, with some peaks in Central Asia.

Almost all mountains in the list are located in the Himalaya and Karakoram ranges to the south and
west of the Tibetan plateau. In fact, all 7,000 m (23,000 ft) peaks in the world are located in the
centre of Asia (East Asia, Central Asia and South Asia)

The highest mountain outside of Asia is Aconcagua (6,962 m or 22,841 ft), which one list has
ranking 189th in the world amongst mountains with a 500 m or 1,640 ft prominence cutoff.

Q Source: General questions: Geography

37 Consider the following statements about Asia Pacific Centre for ICT (APCICT).
1. APCICT is a subsidiary body of the United Nations Economic and Social Commission for Asia and
the Pacific (ESCAP).
2. India has been a member of the Governing Council (GC) of APCICT since its inception.

Select the correct answer using the codes below.


A. 1 only
B. 2 only
C. Both 1 and 2

31
Total Marks : 200
Online Prelims TEST - 28 (TEXTBOOK)
( InsightsIAS Mock Test Series for UPSC Preliminary Exam 2020 ) Mark Scored : 0

D. None of the above

Correct Answer : C

Answer Justification :

Justification: UNESCAP - Asia Pacific Centre for ICT (APCICT):

APCICT is a subsidiary body of the United Nations Economic and Social Commission for Asia
and the Pacific (ESCAP). It came into existence in June 2006 at Incheon, Republic of Korea.
The Centre mission is to strengthen the efforts of the member countries of ESCAP to use ICT
in their socio-economic development through human and institutional capacity building. To
meet this objective, APCICT work is focused on three inter-related pillars. They are; training,
research and advisory services. Together they form an integrated approach to ICT human
capacity building

The centre is aimed at creating ICT-ready environment through capacity-building trainings


and workshops. Its main areas are ICT applications and development; ICT for development
policy, process and governance and e-Government applications

India has been member of the Governing Council (GC) of APCICT since its inception and
attending the GC meetings. The Ministry has been funding UN APCICT to the tune of US$
20,000 per annum. Indian participants have been attending the training
programs/workshops/etc organized by APCICT

Towards cooperation with APCICT, the Ministry (MeITY) intends to extend its expertise
towards development of multi-purpose community e-centers, enhancing Pacific connectivity
for improved ICT access, furthering initiatives in HRD and capacity building, etc

Q Source: https://meity.gov.in/content/multilateral-cooperation

38 Commonwealth Connects Programme (CCP) primarily has a bearing on which of the following sectors
in commonwealth countries?

A. Cultural Heritage
B. Educational exchanges
C. Information and Communication Technology
D. NGOs

Correct Answer : C

Answer Justification :

32
Total Marks : 200
Online Prelims TEST - 28 (TEXTBOOK)
( InsightsIAS Mock Test Series for UPSC Preliminary Exam 2020 ) Mark Scored : 0

Justification: The Commonwealth Connects Programme has laid a clear strategic foundation for
the realisation of its vision of how ICT can be used to promote sustainable development throughout
the Commonwealth and how Commonwealth networks can be used to bridge the digital divide

The programme has taken significant steps in facilitating the sharing of the enormous ICT
resources deployed within the Commonwealth so that ICT-rich member countries do more to
support countries that are ICT-disadvantaged in using ICT for development and improving the
lives of all their citizens

The programme supports projects on a selective basis, looking at their potential for
replication across member states and for demonstrating the potential for collaboration
between Commonwealth governments and other stakeholders

Ministry of Electronics and Information Technology, Government of India is actively involved


in this programme and is providing various inputs based on its strengths in the field of
information and communication and technologies including advice on replication of various
successful projects in the Commonwealth member states

Ministry of Electronics and Information Technology, Government of India has already


facilitated Commonwealth Connects in replicating Indian Hole-in-the-Wall project in Uganda

Q Source: https://meity.gov.in/content/multilateral-cooperation

39 Kiliki language (speech) was invented for which of these movies?

A. Avatar
B. Baahubali
C. Newton
D. Panipat

Correct Answer : B

Answer Justification :

Justification: The new speech was invented for the terrifying warrior tribe called Kalakeya in the
two part Baahubali franchise. It now has evolved into a language with script grammar and more
than 3000 words for everyday communication. It is considered as the world's easiest language.

This fictional language was created by popular lyricist and screenwriter Madhan Karky.

It has 750 words and 40 grammar rules and is written using 22 symbols.

After the success of the movie in 2015 December Singer Smita released 'Baha Kiliki' the first song

33
Total Marks : 200
Online Prelims TEST - 28 (TEXTBOOK)
( InsightsIAS Mock Test Series for UPSC Preliminary Exam 2020 ) Mark Scored : 0

in Kiliki language in YouTube. As of now it was crossed 82 Millions Video Views. Later it was Used
in Baahubali 2: The Conclusion Movie in 2017, which was the sequel of Baahubali: The Beginning

In February 2020 on International Mother Language Day, the film's director SS Rajamouli launched
the official website of Kiliki.

Q Source:
https://www.insightsonindia.com/wp-content/uploads/2020/03/Insights-February-2020-Current-Affai
rs-Compilation.pdf

40 Consider the following about the Foundation for Nutrition (NFN) of the National Dairy Development
Board (NDDB).
1. It seeks to provide nutrition support to children through nutritious products to eradicate
malnutrition.
2. It intends to supply milk and milk products free of cost to Aanganwadis engaged in the Mid-Day
Meal Scheme (MDM).

Select the correct answer using the codes below.


A. 1 only
B. 2 only
C. Both 1 and 2
D. None of the above

Correct Answer : A

Answer Justification :

Justification: NDDB Foundation for Nutrition (NFN) has been promoted by the National Dairy
Development Board (NDDB), its subsidiaries (Indian Immunologicals Limited, IDMC Limited,
Mother Dairy Fruit & Vegetables Private Limited) and Gujarat Cooperative Milk Marketing
Federation (GCMMF).

It has been registered in October 2015 at Anand, Gujarat, as a Society under the Societies
Registration Act 1860 and as a Trust under the Bombay Public Charitable Trust Act as applicable to
the state of Gujarat.

Mission Statement: “NFN would strive to contribute towards eradication of malnutrition in school
children by providing them safe and nutritious products”.

Major Objectives (S2 is incorrect) :

1. To provide nutrition support to children through nutritious products to eradicate malnutrition.

2. To promote consumption of milk and milk products including fortified milk & milk products.

34
Total Marks : 200
Online Prelims TEST - 28 (TEXTBOOK)
( InsightsIAS Mock Test Series for UPSC Preliminary Exam 2020 ) Mark Scored : 0

3. To raise funds and accept donations, grants of money, securities property of any kind from
sources those are conducive to the promotion of the objectives of the society.

4. To receive funds under Corporate Social Responsibility obligations of companies as listed out
in Schedule VII of the Companies Act, 2013 as amended from time to time for the purpose of
providing nutrition support to children.

Q Source: https://www.nfn.org.in/about-us

41 With reference to Indian Council of World Affairs (ICWA), consider the following statements.
1. It is a statutory body.
2. It is an institution of national importance.
3. The President of India is the ex-officio President of ICWA.

Select the correct answer using the codes below.


A. 1 and 2 only
B. 2 only
C. 1 and 3 only
D. 1, 2 and 3

Correct Answer : B

Answer Justification :

Justification: The Indian Council of World Affairs (ICWA) is a New Delhi based Indian think-tank.
Established in 1943, it is devoted exclusively for the study of international relations and foreign
affairs.

Established in 1943 by a group of Indian intellectuals as a think tank.

By an Act of Parliament in 2001, the ICWA was declared an institution of national importance
(it wasn’t established by an act of Parliament, so S1 is wrong).

The Vice President of India is the ex-officio President of ICWA.

It is devoted exclusively for the study of international relations and foreign affairs.

The founder-president of the Council was Sir Tej Bahadur Sapru.

Historic international conferences held by ICWA: ‘Asian Relations Conference’ in 1947 under
the leadership of freedom fighter Sarojini Naidu and ‘United Nations and the New World

35
Total Marks : 200
Online Prelims TEST - 28 (TEXTBOOK)
( InsightsIAS Mock Test Series for UPSC Preliminary Exam 2020 ) Mark Scored : 0

Order’ in 1994.

ICWA has been called by media as one of India's influential think-tanks, but is seen only as an
extension of the Ministry of External Affairs

Q Source: Important organizations from Current affairs

42 Invest India, set up in 2009, is a non-profit venture under the

A. Department for Promotion of Industry and Internal Trade


B. Department of Finance
C. Department of Expenditure
D. Ministry of External Affairs

Correct Answer : A

Answer Justification :

Justification: As the national investment promotion and facilitation agency, Invest India focuses on
sector-specific investor targeting and development of new partnerships to enable sustainable
investments in India. In addition to a core team that focuses on sustainable investments, Invest
India also partners with substantial investment promotion agencies and multilateral organizations.
Invest India also actively works with several Indian states to build capacity as well as bring in global
best practices in investment targeting, promotion and facilitation areas.

Invest India, set up in 2009, is a non-profit venture under the Department for Promotion of Industry
and Internal Trade, Ministry of Commerce and Industry, Government of India.

Q Source: https://www.investindia.gov.in/about-us

43 ‘ReALCraft’ is a work flow based online application system with a bearing on which of these sectors?

A. Fishing
B. Dairy industry
C. Coal washeries
D. Steel

Correct Answer : A

Answer Justification :

Justification: ‘ReALCraft’ is a work flow based online application system for Vessel Registration
under MS Act and License Certificate under MFR Act to the fishing vessels operating along the
Indian coast. It is a national project sanctioned by the Department of Animal Husbandry, Dairying
36
Total Marks : 200
Online Prelims TEST - 28 (TEXTBOOK)
( InsightsIAS Mock Test Series for UPSC Preliminary Exam 2020 ) Mark Scored : 0

and Fisheries, Ministry of Agriculture and Farmers Welfare, Govt. of India for implementing in all
coastal States and UTs.

Objectives

To prepare a National database for fishing vessels.

To prevent Illegal, unregistered and unreported (IUU) vessels in the territorial waters.

To facilitate optimum utilization of the fisheries resources.

To strengthen coastal security.

To strengthen security of fishermen in the sea

Features

Work flow based system.

G2C/ G2G/ G2E/ G2B.

Internet Query.

Enquiry for Citizen.

Services through Internet, Kiosks, Counters and Touch Screen.

SMS Services / Email facility.

Interface for generating engine number for assembled vessels.

Automated Process.

Q Source: http://164.100.150.120/realcraft/web/index.php5

44 Consider the following statements.


1. ‘Depository Receipt’ (DR) may mean a negotiable security issued outside India by a Depository

37
Total Marks : 200
Online Prelims TEST - 28 (TEXTBOOK)
( InsightsIAS Mock Test Series for UPSC Preliminary Exam 2020 ) Mark Scored : 0

bank.
2. DRs can be traded on foreign stock exchanges.

Select the correct answer using the codes below.


A. 1 only
B. 2 only
C. Both 1 and 2
D. None of the above

Correct Answer : C

Answer Justification :

Justification: Statement 1: ‘Depository Receipt’ (DR) means a negotiable security issued outside
India by a Depository bank, on behalf of an Indian company, which represent the local Rupee
denominated equity shares of the company held as deposit by a Custodian bank in India. DRs are
traded on Stock Exchanges in the US, Singapore, Luxembourg, etc. DRs listed and traded in the US
markets are known as American Depository Receipts (ADRs) and those listed and traded
anywhere/elsewhere are known as Global Depository Receipts (GDRs). DRs are governed by
Notification No. FEMA 330/ 2014-RB, issued by Reserve bank of India

Learning: “Employees’ Stock Option” means the option given to the directors, officers or
employees of a company or of its holding company or joint venture or wholly owned overseas
subsidiary/subsidiaries, if any, which gives such directors, officers or employees, the benefit or right
to purchase, or to subscribe for, the shares of the company at a future date at a pre-determined
price.

Q Source: Basics: FEMA

45 Consider the following about World Economic Forum’s (WEF) Global Smart Cities Alliance on
Technology Governance.
1. It is an exclusive forum of the Group of 20 (G20) and Commonwealth Sustainable Cities Network.
2. The alliance seeks to create global norms and policy standards for the use of connected devices in
public spaces

Select the correct answer using the codes below.


A. 1 only
B. 2 only
C. Both 1 and 2
D. None of the above

Correct Answer : B

Answer Justification :

Context and Justification: India has joined the World Economic Forum’s (WEF) G20 Global Smart

38
Total Marks : 200
Online Prelims TEST - 28 (TEXTBOOK)
( InsightsIAS Mock Test Series for UPSC Preliminary Exam 2020 ) Mark Scored : 0

Cities Alliance on Technology Governance.

India’s joining the league is a critical first step towards accelerating global best practices, fostering
greater openness and public trust as well as mitigating risks regarding the collection of data in
public spaces.

Statement 1: The Global Smart Cities Alliance's founding set of institutional partners include the
presidents and host nations of the Group of 20 (G20) in 2019 and 2020; Japan and the Kingdom of
Saudi Arabia; the Smart City Mission of India; Cities for All; Cities Today Institute; Commonwealth
Local Government Forum; Commonwealth Sustainable Cities Network among others.

Statement 2: What is it?

It is a league of 15 of world’s leading city networks and technology governance organisations that
will work towards advancing responsible and ethical use of smart city technologies.

Established in June 2019, in conjunction with G20 Summit held in Osaka, Japan.

It seeks to create global norms and policy standards for the use of connected devices in public
spaces.

The aim of the alliance is to promote the responsible and ethical use of smart city technologies by
establishing global norms and policy standards for the use of connected devices in public spaces.

Q Source: In news

46 Consider the following statements about Bhitarkanika protected area.


1. It has one of the largest populations of endangered saltwater crocodile in India.
2. It hosts the second largest mangrove ecosystem in India.

Which of the above is/are correct?


A. 1 only
B. 2 only
C. Both 1 and 2
D. None of the above

Correct Answer : C

Answer Justification :

Learning: Bhitarkanika National Park is a national park located in Kendrapara district of Odisha in
eastern India.

Gahirmatha Beach and Marine Sanctuary lies to the east, and separates swamp region cover with
canopy of mangroves from the Bay of Bengal.

The national park is home to Saltwater crocodile (Crocodylus porosus), Indian python, King cobra,
black ibis, darters and many other species of flora and fauna. It hosts a large number of mangrove

39
Total Marks : 200
Online Prelims TEST - 28 (TEXTBOOK)
( InsightsIAS Mock Test Series for UPSC Preliminary Exam 2020 ) Mark Scored : 0

species, and is the second largest mangrove ecosystem in India.

The national park and wildlife sanctuary is inundated by the rivers Brahmani, Baitarani, Dhamra,
Pathsala.

Bhitarkanika has one of the largest populations of endangered saltwater crocodile in India and is
globally unique in that, 10% of the adults exceed 6 m length.

Q Source: Protected Areas

47 Match the following tribes with the states they are found in.
1. Maldhari: Gujarat
2. Dhuruba: Odisha
3. Kolha: Nagaland

Select the correct answer using the codes below.


A. 1 only
B. 1 and 2 only
C. 2 and 3 only
D. 1 and 3 only

Correct Answer : B

Answer Justification :

Justification: Statement 1: The Maldhari community is a tribe of herdsmen in the border state of
Gujarat. The name Maldhari means owner of goods - in this case, goods referring to cattle. The
Maldharis have lived in the Gir National Park, in the Banni Grasslands Reserve area, for the past
thousand years.

Statement 2 and 3: Dhuruba is a tribe of Odisha.

Kolha is numerically the most important tribe of Keonjhar District, in Odisha. Most of them reside in
the Bhuyan Hills and the adjoining areas of Nayagarh. Kolhas have a separate language of their
own, but most of them use Oriya and Hindi as a subsidiary language.

Q Source: https://tribal.nic.in/repository/

48 Which of these movements was also known as the “Vande Matram movement”?

A. Swadeshi movement
B. Non-cooperation movement
C. Civil disobedience movement
D. Quit India Movement

40
Total Marks : 200
Online Prelims TEST - 28 (TEXTBOOK)
( InsightsIAS Mock Test Series for UPSC Preliminary Exam 2020 ) Mark Scored : 0

Correct Answer : A

Answer Justification :

Learning: Option A is correct.

L. M. Bhole identifies five phases of the Swadeshi movement.

1850 to 1904: developed by leaders like Dadabhai Naoroji, Gokhale, Ranade, Tilak, G.V. Joshi
and Bhaswat.K.Nigoni. This was also known as First Swadeshi Movement.

1905 to 1917: Began with and because of the partition of Bengal in 1905 by Lord Curzon.

1918 to 1947: Swadeshi thought shaped by Gandhi, accompanied by the rise of Indian
industrialists.

1948 to 1991: Widespread curbs on international and inter-state trade. India became a
bastion of obsolete technology during the licence-permit raj.

1991 onwards: liberalization and globalization. Foreign capital, foreign technology, and many
foreign goods are not excluded and doctrine of export-led growth resulted in modern
industrialism.

The second Swadeshi movement started with the partition of Bengal by the Viceroy of India, Lord
Curzon in 1905 and continued up to 1911. It was the most successful of the pre-Gandhian
movement.

Q Source: Based on UPSC past year papers

49 The GEMINI device, developed by Indian National Centre for Ocean Information Services (INCOIS),
seen in news sometimes
1. receives and transfers the data received from GAGAN satellite/s to a mobile device
2. helps to provide information related to disaster warnings when fishermen move away from the coast
beyond a certain range

Select the correct answer using the codes below.


A. 1 only
B. 2 only
C. Both 1 and 2
D. None of the above

Correct Answer : C

41
Total Marks : 200
Online Prelims TEST - 28 (TEXTBOOK)
( InsightsIAS Mock Test Series for UPSC Preliminary Exam 2020 ) Mark Scored : 0

Answer Justification :

Justification: It is a device that used for seamless and effective dissemination of emergency
information and communication on disaster warnings, Potential Fishing Zones (PFZ) and Ocean
States Forecasts (OSF) to fishermen.

The device will help to provide information related to disaster warnings when fishermen move away
from the coast beyond 10 to 12 kilometres.

The GEMINI device receives and transfers the data received from GAGAN satellite/s to a mobile
through Bluetooth communication. A mobile application developed by INCOIS decodes and displays
the information in nine regional languages.

It has been developed by Indian National Centre for Ocean Information Services (INCOIS), and
Airports Authority of India (AAI).

It is electronically designed and manufactured by a private industry M/S Acord, Bangalore under
Make in India Program.

Q Source: In news

50 The setting up of Gram Nyayalayas is considered as an important measure to reduce arrears and is a
part of the judicial reforms. Consider the following about them.
1. The Gram Nyayalayas are presided over by a Magistrate who enjoys the same salary and benefits as
of a High Court Judge,
2. The Court can function as a mobile court at any place within the jurisdiction of such Gram
Nyayalaya.
3. They have both civil and criminal jurisdiction over the offences.
4. Gram Nyayalayas has been given power to accept certain evidences which would otherwise not be
acceptable under Indian Evidence Act.
5. Any appeal from the decision of a Gram Nyayalaya lies straight to the High Court.

Select the correct answer using the codes below.


A. 2, 3, 4 and 5 only
B. 1 and 5 only
C. 2, 3 and 4 only
D. 1, 2, 3, 4 and 5

Correct Answer : C

Answer Justification :

Justification: Gram Nyayalayas or village courts are established under the Gram Nyayalayas Act,
2008 for speedy and easy access to justice system in the rural areas of India. The Act came into
force from 2 October 2009.

Statement 1-4: The Gram Nyayalayas are presided over by a Nyayadhikari, who will have the same

42
Total Marks : 200
Online Prelims TEST - 28 (TEXTBOOK)
( InsightsIAS Mock Test Series for UPSC Preliminary Exam 2020 ) Mark Scored : 0

power, enjoy same salary and benefits of a Judicial Magistrate of First Class. Such Nyayadhikari are
to be appointed by the State Government in consultation with the respective High Court.

Jurisdiction:

• A Gram Nyayalaya have jurisdiction over an area specified by a notification by the State
Government in consultation with the respective High Court.

• The Court can function as a mobile court at any place within the jurisdiction of such Gram
Nyayalaya, after giving wide publicity to that regards.

• They have both civil and criminal jurisdiction over the offences.

• The pecuniary jurisdiction of the Nyayalayas are fixed by the respective High Courts.

Gram Nyayalayas has been given power to accept certain evidences which would otherwise not be
acceptable under Indian Evidence Act.

Statement 5: Appeal in criminal cases shall lie to the Court of Session, which shall be heard and
disposed of within a period of six months from the date of filing of such appeal. Appeal in civil cases
shall lie to the District Court, which shall be heard and disposed of within a period of six months
from the date of filing of the appeal.

Learning: So far only 11 states have taken steps to notify Gram Nyayalayas. Several states have
issued notifications for establishing ‘Gram Nyayalayas’ but all of them were not functioning except
in Kerala, Maharashtra and Rajasthan. Only 208 ‘Gram Nyayalayas’ are functioning in the country
as against 2,500 estimated to be required by the 12th five-year plan.

It is estimated that Gram Nyayalayas can reduce around 50% of the pendency of cases in
subordinate courts and can take care of the new litigations which will be disposed within six
months.

Q Source:
https://www.insightsonindia.com/wp-content/uploads/2020/03/Insights-February-2020-Current-Affai
rs-Compilation.pdf

51 Consider the following statements.


1. Curzon brought in the Indian Universities Act of 1904, which brought all the universities in India
under the control of the government.
2. During Curzon's times, the Indian Official Secrets Act (1904) was enacted mainly to restrict the
freedom of the press.

Select the correct answer using the codes below.


A. 1 only
B. 2 only
C. Both 1 and 2
D. None of the above

43
Total Marks : 200
Online Prelims TEST - 28 (TEXTBOOK)
( InsightsIAS Mock Test Series for UPSC Preliminary Exam 2020 ) Mark Scored : 0

Correct Answer : C

Answer Justification :

Justification: The immediate cause for the rise of extremism in India is said to be the reactionary
rule of Lord Curzon.

S2: The Indian Official Secrets Act, 1904 was enacted during the time of Lord Curzon, Viceroy of
India from 1899 to 1905.

One of the main purposes of the Act was to muzzle the voice of nationalist publications.

The Indian Official Secrets Act (Act No XIX of 1923) replaced the earlier Act, and was extended to
all matters of secrecy and confidentiality in governance in the country.

The secrecy law broadly deals with two aspects — spying or espionage, which is dealt with in
Section 3 of the Act, and disclosure of other secret information of the government, which is dealt
with in Section 5. The secret information can be any official code, password, sketch, plan, model,
article, note, document or information.

The Sedition Act and the Official Secrets Act reduced the freedoms of all people.

S1: Curzon took a serious view of the fall in the standard of education and discipline in the
educational institutions.

In his view the universities had degenerated into factories for producing political
revolutionaries. To set the educational system in order, he instituted in 1902, a Universities
Commission to go into the entire question of university education in the country.

On the basis of the findings and recommendations of the Commission, Curzon brought in the
Indian Universities Act of 1904, which brought all the universities in India under the control
of the government.

He passed the Calcutta Corporation Act, (1899) reducing the Indian control of this local body.

Q Source: 12th TamilNadu History Textbook

52 Ministries or departments of the Union Government can be merged by

A. Executive action by the Cabinet


B. Legislative action only
C. Constitutional amendment only
D. A mutual agreement between the Ministers of the concerned departments

44
Total Marks : 200
Online Prelims TEST - 28 (TEXTBOOK)
( InsightsIAS Mock Test Series for UPSC Preliminary Exam 2020 ) Mark Scored : 0

Correct Answer : A

Answer Justification :

Learning: The Govt. of India (Allocation of Business) Rules, 1961 are made by the President of
India under Article 77 of the Constitution for the allocation of business of the Govt. of India.

The Ministries/Departments of the Govt. are created (or merged, dissolved) by the President on the
advice of the Prime Minister under these Rules.

The Cabinet Secretariat is under the direct charge of the Prime Minister. Any major decision of the
Cabinet Secretariat is actually the decision of the Cabinet lead by the PM.

Q Source: General issues: Indian polity

53 Which of these acts provided for the establishment of an All India Federation (which never came into
being) to be based on a Union of the provinces of British India and the Princely States?

A. The Government of India Act of 1935


B. Act of 1919
C. Act of 1909
D. The Independence of India Act of 1947

Correct Answer : A

Answer Justification :

Justification: The proposed all India federation included 11 provinces of British India, 6 Chief
Commissioners Provinces and those princely states who might accede to the federation.

The term on which a state joined the Federation were to be laid down in the Instrument of
Accession.

Joining the federation was compulsory for the British Provinces and chief commissioner’s
provinces.

For princely states, the accession to the Federation was voluntary.

The federation could not be established until a number of states, the rulers whereof were
entitled to choose not less than half of the 104 seats of the council of state, and the aggregate
population whereof amounted to be at least one half of the total population of all the Indian
stales had acceded to the federation.

Princely states never joined, and thus the federation never came into being. So, (a) is correct.
45
Total Marks : 200
Online Prelims TEST - 28 (TEXTBOOK)
( InsightsIAS Mock Test Series for UPSC Preliminary Exam 2020 ) Mark Scored : 0

Q Source: Past year papers: UPSC

54 Which of these states recently passed a bill declaring a river delta as a Protected Special Agricultural
Zone (PSAZ)?

A. Kerala
B. Karnataka
C. Tamil Nadu
D. Maharashtra

Correct Answer : C

Answer Justification :

Justification: The new act was passed by Tamil Nadu assembly in February, 2020, declaring Kaveri
delta region as a Protected Special Agriculture Zone (PSAZ).

The protected zone of cauvery delta will include Thanjavur, Tiruvarur, Nagapattinam districts and
certain other delta regions. It is meant to offer farmers of the delta region relief from further
hydrocarbon exploration.

PSAZ is aimed at protecting the Cauvery delta region for the future, fulfilling TN’s food
requirements and ensuring the welfare of delta farmers. It has recognised farmer concerns about
hydrocarbon exploration and accorded primacy to food security.

Need for: The delta, which produces 33 lakh tonnes of grains in 28 lakh acres, has seen multiple
protests for a decade over methane, hydrocarbon, oil and natural gas projects, which required
acquisition of fertile lands and well drilling — proposals which triggered fears of groundwater
contamination.

Read more about the law here:


https://www.thehindu.com/news/national/tamil-nadu/protected-special-agricultural-zone-in-tamil-na
dus-cauvery-delta-experts-seek-powers-to-implementing-authority/article30954023.ece

Q Source:
https://www.insightsonindia.com/wp-content/uploads/2020/03/Insights-February-2020-Current-Affai
rs-Compilation.pdf

55 Consider the following statements.


1. Organoids are a group of cells grown in laboratories into three-dimensional, miniature structures
that mimic the cell arrangement of a fully-grown organ.
2. Organoids achieve all the functional maturity of human organs.
3. Organoids are useful in precision medicine since they can be used to study the safety and efficacy of
new drugs and also test the response of tissues to existing medicines.

Select the correct answer using the codes below.

46
Total Marks : 200
Online Prelims TEST - 28 (TEXTBOOK)
( InsightsIAS Mock Test Series for UPSC Preliminary Exam 2020 ) Mark Scored : 0

A. 1 and 3 only
B. 3 only
C. 2 only
D. 1, 2 and 3

Correct Answer : A

Answer Justification :

Justification: Statement 1: Organoids are a group of cells grown in laboratories into three-
dimensional, miniature structures that mimic the cell arrangement of a fully-grown organ.

They are tiny (typically the size of a pea) organ-like structures that do not achieve all the functional
maturity of human organs but often resemble the early stages of a developing tissue.

Most organoids contain only a subset of all the cells seen in a real organ, but lack blood vessels to
make them fully functional.

Grown in the lab using stem cells that can become any of the specialised cells seen in the human
body, or stem cells taken from the organ or adults cells that have been induced to behave like stem
cells, scientifically called induced pluripotent stem cells (iPSC). Organoids of the brain, small
intestine, kidney, heart, stomach, eyes, liver, pancreas, prostate, salivary glands, and inner ear to
name a few have already been developed in the laboratory.

Statement 2: Organoids offer new opportunities to studying proteins and genes that are critical for
the development of an organ. This helps in knowing how a mutation in a specific gene causes a
disease or disorder.

Statement 3: Organoids can be used to study the safety and efficacy of new drugs and also test the
response of tissues to existing medicines. Organoids will bring precision medicine closer to reality
by developing patient-specific treatment strategies by studying which drugs the patient is most
sensitive to.

Q Source:
https://www.insightsonindia.com/wp-content/uploads/2019/11/Insights-October-2019-Current-Affair
s-Compilation.pdf

56 Who among the following renounced the title of ‘Knighthood’ due to the Jalianwala Bagh incidents of
1919?

A. Sarojini Naidu
B. Gangadhar Tilak
C. Rabindranath Tagore
D. Maulana Abul Kalam Azaad

Correct Answer : C

47
Total Marks : 200
Online Prelims TEST - 28 (TEXTBOOK)
( InsightsIAS Mock Test Series for UPSC Preliminary Exam 2020 ) Mark Scored : 0

Answer Justification :

Justification: Rabindranath Tagore discarded this Title of Knighthood for the inhuman act of
British Government in Jallianwalabag in 1919.

Q Source: Major events linked with personalities: Modern India

57 Which of the following scheme provides for financial assistance to patients, living below poverty line
and suffering from major life threatening diseases to receive medical treatment at any of the super
specialty Government hospitals?

A. Rashtriya Swasthya Bima Yojana (RSBY)


B. Central Government Health Scheme (CGHS)
C. Rashtriya Arogya Nidhi
D. Pradhan Mantri Swasthya Suraksha Karyakram (PMSSK)

Correct Answer : C

Answer Justification :

Learning: Poor Patients in India get financial Support under the following schemes:

Health Ministers Discretionary Grant (HMDG)

RAN (Health Ministers Cancer Patient Fund)

Rashtriya Arogya Nidhi

The Rashtriya Arogya Nidhi (RAN) was set up by a resolution and registered under the Societies
Registration Act, 1860, as a Society.

The financial assistance to such patients is released in the form of ‘one-time grant’, which is
released to the Medical Superintendent of the Hospital in which the treatment has been/is being
received.

Under RAN Revolving Funds have been set up in 13 Central Government Hospitals/Institutions
located all over India for providing financial assistance for treatment up to Rs. 2 lakh.

In addition financial assistance is provided for individual cases referred by Government


hospitals/institutions, which do not have a Revolving Fund and for cases referred by 13 Government
hospitals/institutions with Revolving Funds for assistance exceeding Rs. 2 lakh.

Q Source: https://mohfw.gov.in/major-programmes/poor-patients-financial-support

48
Total Marks : 200
Online Prelims TEST - 28 (TEXTBOOK)
( InsightsIAS Mock Test Series for UPSC Preliminary Exam 2020 ) Mark Scored : 0

58 Theenperayam or the great assembly (perayam), as mentioned in Sangam texts, consisted of

A. Government officers
B. Poets
C. Sculpture makers
D. Temple priests

Correct Answer : A

Answer Justification :

Learning: The administrative machinery of the age was described by the Sangam texts.

The policies of the king were controlled by a system of checks and balances in the councils.
Silappadikaram refers to the two types of councils —Aimperunkulu and Enperayam.

The aimperunkulu or the council of five members was the council of the ministers.
Theenperayam or the great assembly (perayam) consisted of 8 members (government
officers).

This worked as an administrative machinery of the state. These two assemblies that of the
Five and that of the Eight functioned as administrative bodies, though their function was
generally advisory in character. However, their advice was rarely rejected by the king.

Their important function was judicial though the aimperunkulu seems to have been solely in
chaRige of it as described by Maduraikkanji.

It is important to note that in spite of all the glory attached to the ancient king, the ethos of
Indian administration has been in the direction of limited or popular monarchy.

This can be observed in South India from very early times even more than in the north and
each followed its own model of administration. Every local unit, however small and in
whatever corner it was situated, was administered by a local assembly.

The avai and the manram are the terms used for this unit in Sangam works. Such assembly is
commonly referred to as arankuravaiyam, which were known for its just decision. These can
be taken to be the forerunner of our modern panchayat.

Q Source: Based on past year UPSC papers

59 Classical Swine Fever (CSF) is caused mainly in

49
Total Marks : 200
Online Prelims TEST - 28 (TEXTBOOK)
( InsightsIAS Mock Test Series for UPSC Preliminary Exam 2020 ) Mark Scored : 0

A. Birds
B. Cattle
C. Pigs and sheep
D. None of the above

Correct Answer : C

Answer Justification :

Justification: Hog Cholera or Classical swine fever (CSF) is a contagious viral disease of domestic
and wild swine. It happens due to the viruses that bring viral diarrhea in pigs and ailments in sheep.

The disease does not harm humans but all-important precautions are advised to follow.

Concerns for India: Classical Swine Fever (CSF) is one of the biggest pigs’ diseases in India. It
causes a loss of about 400 crores of rupees per year in India. This has led to a decrease in the
population of pigs in 2019.

Learning: India currently requires 22 million doses of the CSF (Classical Swine Fever) vaccine
every year. However, currently, only 1.2 million doses are being produced. The reason behind its
less production is that only 50 doses can be prepared from the spleen of a rabbit.

The Indian Institute of Veterinary Research (IVRI) of the Indian Council of Agricultural Research
(ICAR) has developed a new vaccine to control classical swine fever.

About the vaccine and it’s significance: It is a live attenuated CSF cell culture vaccine (indigenous
strain). The indigenously developed vaccine will help in saving rabbits as the currently used vaccine
(lapinized CSF vaccine) is produced by sacrificing large numbers of rabbits.

Besides, the new vaccine gives immunity for two years as compared to 3 to 6 months protection
under the currently used vaccines. The new vaccine will be a part of the government’s One Health
Initiative.

Q Source:
https://www.insightsonindia.com/wp-content/uploads/2020/03/Insights-February-2020-Current-Affai
rs-Compilation.pdf

60 The Gender Parity Index (GPI) is released by

A. World Economic Forum (WEF)


B. World Bank
C. UNESCO
D. UN Commission on Women

Correct Answer : C

50
Total Marks : 200
Online Prelims TEST - 28 (TEXTBOOK)
( InsightsIAS Mock Test Series for UPSC Preliminary Exam 2020 ) Mark Scored : 0

Answer Justification :

Learning: In its simplest form, it is calculated as the quotient of the number of females by the
number of males enrolled in a given stage of education (primary, secondary, etc.).

It is used by international organizations, particularly in measuring the progress of developing


countries.

The Institute for Statistics of UNESCO also uses a more general definition of GPI: for any
development indicator one can define the GPI relative to this indicator by dividing its value for
females by its value for males.

For example, some UNESCO documents consider gender parity in literacy by dividing the female
value of an indicator by the male value of the same indicator.

GPI equal to 1 indicates parity between females and males. In general, a value less than 1 indicates
a disparity in favour of boys and a value greater than 1 indicates a disparity in favour of girls.

Limitations: This index does not show whether improvement or regression is due to the
performance of one of the gender groups (boys or girls). Interpretation of the GPI requires trend
analysis of the underlying indicators.

The Union Finance Minister released Gender Parity Index specific to India sometime back.

The main aim of developing Gender Parity Index commissioned jointly by FLO and FICCI is to
evaluate gender diversity and empowerment of women in the formal sector and the progress made
over the years.

Q Source: http://uis.unesco.org/en/glossary-term/gender-parity-index-gpi

61 Some of the potential applications of the operational NavIC system include


1. Wagon Tracking for Railways
2. Vehicle tracking System for State Transport Buses, Taxis and all public service vehicles
3. Disaster management

Select the correct answer using the codes below.


A. 1 only
B. 2 and 3 only
C. 3 only
D. 1, 2 and 3

Correct Answer : D

Answer Justification :

Justification: The Indian Regional Navigation Satellite System (IRNSS), with an operational name
of NavIC (Navigation with Indian Constellation), is a completely home-grown solution of ISRO which
enables navigation in the Indian Ocean region using a combination of GEO and GSO satellites.

51
Total Marks : 200
Online Prelims TEST - 28 (TEXTBOOK)
( InsightsIAS Mock Test Series for UPSC Preliminary Exam 2020 ) Mark Scored : 0

The NavIC system has been designed to provide localisation fixes (with a resolution of 20 m or
better) over the Indian sub-continent – in two forms: a Standard Positioning Service (SPS) for all
civilian users, and a higher precision Restricted Service (RS) for authorized users.

Some of the immediate applications for an operational NavIC system are:

Railways: Wagon Tracking,

Vehicle tracking System for State Transport Buses, Taxis and all public service vehicles

Tracking of fishing vessels

Tracking and geo-fencing of mechanised vehicles in service

Disaster management

Applications of Satellite based Navigation Systems Technological advancement in the design of


ASICs has resulted in the development of cheap, low-power ground based receivers, and their
associated circuitry.

There has been considerable miniaturisation of this hardware, which has allowed the deployment of
these devices in small form-factor, portable devices which enable distributed, on-the-go localisation.
This information has a lot of business potential, and thus the development of an indigenous system -
dovetails well with ‘Make in India’, ‘Start up India’, the ‘Smart Cities’ and ‘Digital India’
programs.

Q Source: https://meity.gov.in/development-navic-chip-commercialcivilian-purposes

62 Consider the following statements.


1. Despite the constitutional privilege of free speech for a Member of Parliament (MP) in the House,
whatever an MP says in the house is subject to the Rules of Parliament and the control of
proceedings by the Speaker.
2. If the Speaker is of opinion that words have been used in a House debate which are
unparliamentary, the Speaker may order that such words be expunged from the proceedings of the
House.

Select the correct answer using the codes below.


A. 1 only
B. 2 only
C. Both 1 and 2
D. None of the above

52
Total Marks : 200
Online Prelims TEST - 28 (TEXTBOOK)
( InsightsIAS Mock Test Series for UPSC Preliminary Exam 2020 ) Mark Scored : 0

Correct Answer : C

Answer Justification :

Justification: Statement 1: Despite Article 105(2) of the constitution, Whatever an MP says is


subject to the discipline of the Rules of Parliament, the “good sense” of Members, and the control of
proceedings by the Speaker. These checks ensure a dignified debate in the house.

Statement 2: Rule 380 (“Expunction”) of the Rules of Procedure and Conduct of Business in Lok
Sabha says: “If the Speaker is of opinion that words have been used in debate which are defamatory
or indecent or unparliamentary or undignified, the Speaker may, while exercising discretion order
that such words be expunged from the proceedings of the House.”

Rule 381 says: “The portion of the proceedings of the House so expunged shall be marked by
asterisks and an explanatory footnote shall be inserted in the proceedings as follows: ‘Expunged as
ordered by the Chair’.”

Q Source: In news

63 Consider the following statements.


1. Russia borders both Black Sea and Mediterranean Sea.
2. Georgia borders both Caspian Sea and Black Sea.

Select the correct answer using the codes below.


A. 1 only
B. 2 only
C. Both 1 and 2
D. None of the above

Correct Answer : D

Answer Justification :

Justification:

53
Total Marks : 200
Online Prelims TEST - 28 (TEXTBOOK)
( InsightsIAS Mock Test Series for UPSC Preliminary Exam 2020 ) Mark Scored : 0

Q Source: Map-based questions: West Asia

64 Consider the following statements about provisions under the Sixth Schedule.
1. There shall be a District Council for each autonomous district of which all the members shall be
elected on the basis of adult suffrage.
2. The Governor can modify / divide the boundaries of the said Tribal areas (district) by a notification.

Select the correct answer using the codes below.


A. 1 only
B. 2 only
C. Both 1 and 2
D. None of the above

Correct Answer : B

Answer Justification :

Justification: Statement 1: Constitution of District Councils and Regional Councils:

(1) There shall be a District Council for each autonomous district consisting of not more than thirty
members, of whom not more than four persons shall be nominated by the Governor and the
rest shall be elected on the basis of adult suffrage.

54
Total Marks : 200
Online Prelims TEST - 28 (TEXTBOOK)
( InsightsIAS Mock Test Series for UPSC Preliminary Exam 2020 ) Mark Scored : 0

(2) There shall be a separate Regional Council for each area constituted an autonomous region.

(3) Each District Council and each Regional Council shall be a body corporate by the name
respectively of the District Council of (name of district) and the Regional Council of (name of
region), shall have perpetual succession and a common seal and shall by the said name sue and be
sued.

Statement 2: As per the Sixth Schedule, the four states viz. Assam, Meghalaya, Tripura and
Mizoram contain the Tribal Areas which are technically different from the Scheduled Areas. Though
these areas fall within the executive authority of the state, provision has been made for the creation
of the District Councils and regional councils for the exercise of the certain legislative and judicial
powers. Each district is an autonomous district and Governor can modify / divide the
boundaries of the said Tribal areas (district) by a notification.

65 They introduced the art of excavating temples from the rock, and the Dravidian style of temple
architecture began with their rule. They were?

A. Cheras
B. Pallavas
C. Eastern Chalukyas
D. Jaffna

Correct Answer : B

Answer Justification :

Learning: Pallava age was a great age of temple building.

It was a gradual evolution starting from the cave temples to monolithic rathas and culminated in
structural temples.

The development of temple architecture under the Pallavas can be seen in several stages, which
started with Mahendravarman I introducing rock-cut temples.

This style of Pallava temples are seen at places like Mandagappattu, Mahendravadi, Mamandur,
Dalavanur, Tiruchirappalli, Vallam, Siyamangalam and Tirukalukkunram.

Q Source: Page 127: TN 11th Standard History Textbook

66 Triangular Cooperation (TrC), a term that is often seen in news, is used to denote cooperation
between

A. Developed countries, NGOs and Inter-governmental organizations (IGO)


B. Developing countries, Least Developed countries and Community Organizations
C. Two or more developing countries in collaboration with a third partner such as an IGO
D. Two or more developed countries in collaboration with a third partner such as an IGO

55
Total Marks : 200
Online Prelims TEST - 28 (TEXTBOOK)
( InsightsIAS Mock Test Series for UPSC Preliminary Exam 2020 ) Mark Scored : 0

Correct Answer : C

Answer Justification :

Justification: South-South Cooperation is the mutual sharing and exchange of key development
solutions – knowledge, experiences and good practices, policies, technology, and resources –
between and among countries in the global south.

Triangular Cooperation (TrC) refers to partnerships between two or more developing countries
in collaboration with a third partner, typically a developed country/traditional donor, emerging
economy and/or multilateral organization to share key development solutions – knowledge, capacity,
expertise, experiences and good practices, policies, technology and resources.

Example of TrC: In 2011, an FAO-facilitated SSC exchange was launched to improve the
aquaculture sector in Namibia through aquafeed production, species diversification and the
development of aquaculture in dry lands. Viet Nam supplied three long-term experts and five
technicians who were supported by funding from the Government of Spain

Learning: There is a growing consensus around the world, affirming South-South and Triangular
Cooperation as effective instruments for catalyzing agricultural development. South-South
Cooperation (SSC) is widely accepted as a complementary model of development cooperation to the
traditional NorthSouth model, in overcoming common challenges and achieving food security,
poverty reduction and the sustainable management of natural resources.

Q Source: http://www.fao.org/3/a-h0045e.pdf

67 Once the capital of the Pallava dynasty, it was also the home of the famous 6th century CE poet
Bharavi and the famous 11th to 12th century CE philosopher Ramanuja. The city was

A. Kanchipuram
B. Ariyayur
C. Mamallapuram
D. Kalindi

Correct Answer : A

Answer Justification :

Justification: Once a capital of the Pallava dynasty, Kanchipuram was also a noted centre of
learning for Tamil and Sanskrit scholars. Dinganaga, a Buddhist writer came to study at Kanchi.

It hosts many temples dedicated to Shiva and Vishnu.

Kanchi was also the home of the famous 6th century CE poet Bharavi who wrote the
Kiratarjuniya and the famous 11th to 12th century CE Hindu philosopher Ramanuja.

56
Total Marks : 200
Online Prelims TEST - 28 (TEXTBOOK)
( InsightsIAS Mock Test Series for UPSC Preliminary Exam 2020 ) Mark Scored : 0

Still today an important religious centre, the site has over hundred temples and is also noted
for its production of fine silk saris.

Q Source: Additional Research: Page 118: TN 11th Standard Textbook

68 Consider the following about Agroecology.


1. It is based on applying ecological concepts and principles to optimize interactions between plants,
animals, humans and the environment.
2. As per FAO, agroecological producers recognize that a highly successful methodology for promoting
farmer innovation is a strict patent system and not a traditional farmer-to-farmer learning approach.
3. Agroecology looks for global solutions to local problems and aims for linkages of local farm with the
global markets.

Select the correct answer using the codes below.


A. 1, 2 and 3
B. 1 only
C. 1 and 3 only
D. 2 and 3 only

Correct Answer : B

Answer Justification :

Justification: Statement 1: Agroecology is based on applying ecological concepts and principles to


optimize interactions between plants, animals, humans and the environment while taking into
consideration the social aspects that need to be addressed for a sustainable and fair food system.

By building synergies, agroecology can support food production and food security and nutrition
while restoring the ecosystem services and biodiversity that are essential for sustainable
agriculture. Agroecology can play an important role in building resilience and adapting to climate
change.

Agroecology is based on context-specific design and organization, of crops, livestock, farms and
landscapes. It works with solutions that conserve above and below ground biodiversity as well as
cultural and knowledge diversity with a focus on women’s and youth’s role in agriculture.

Statement 2 and 3: Agroecological producers recognize that a highly successful methodology for
promoting farmer innovation is farmer-to-farmer learning and sharing.

Agroecology looks for local solutions and linkages with the local economy and local markets, and
keeps farmers in the field with improved livelihoods and a better quality of life.

Q Source: http://www.fao.org/agroecology/overview/en/

69 The strength of legislative assembly in a state is fixed by

57
Total Marks : 200
Online Prelims TEST - 28 (TEXTBOOK)
( InsightsIAS Mock Test Series for UPSC Preliminary Exam 2020 ) Mark Scored : 0

A. President of India
B. Parliament, on a case to case basis
C. Population of the state as provided in the Constitution
D. Governor of the State

Correct Answer : C

Answer Justification :

Justification: This is based on the population of the state. A state with a larger population like UP
will have more seats in the assembly.

The Constitution of India states that a State Legislative Assembly must have no less than 60 and no
more than 500 members however an exception may be granted via an Act of Parliament as is the
case in the states of Goa, Sikkim, Mizoram and the union territory of Puducherry which have fewer
than 60 members.

Q Source: Basics of Polity

70 Hiuen Tsang portray the social life in the times of Harsha. Consider the following with reference to it.
1. He mentions that certain privileged sections of society received land grants from kings.
2. He mentions that the Sudras practiced agriculture.
3. He notes that women enjoyed high status in literary societies and remarriage of widows was
permitted.

Select the correct answer using the codes below.


A. 1 only
B. 2 and 3 only
C. 3 only
D. 1 and 2 only

Correct Answer : D

Answer Justification :

The fourfold division of the society – Brahmin, Kshatriya, Vysya and Sudra – was prevalent.

The Brahmins were the privileged section of the society and they were given land grants by the
kings. Hiuen Tsang mentions that the Sudras practiced agriculture.

Learning: Hiuen Tsang mentions three ways of disposal of the dead – cremation, water burial and
exposure in the woods.

The position of women was not satisfactory. The institution of Swyamvara (the choice of choosing
her husband) had declined.

58
Total Marks : 200
Online Prelims TEST - 28 (TEXTBOOK)
( InsightsIAS Mock Test Series for UPSC Preliminary Exam 2020 ) Mark Scored : 0

Remarriage of widows was not permitted, particularly among the higher castes. The system of
dowry had also become common. The practice of sati was also prevalent.

Trade and commerce had declined during Harsha’s period, which is evident from the decline of
trade centres, less number of coins, and slow activities of merchant guilds.

The decline of trade in turn affected the handicrafts industry and agriculture.
Since there was no large-scale demand for goods, the farmers began to produce only in a
limited way. This led to the rise of self-sufficient village economy.

Q Source: Page 117: TN 11th Standard Textbook

71 IFAW (International Fund for Animal Welfare)


1. is a non-profit organization providing hands-on assistance to animals
2. was founded by the World Wildlife Fund (WWF)

Select the correct answer using the codes below.


A. 1 only
B. 2 only
C. Both 1 and 2
D. None of the above

Correct Answer : A

Answer Justification :

Justification: The International Fund for Animal Welfare (IFAW) is one of the largest animal
welfare and conservation charities in the world.

The group's declared mission is to "rescue and protect animals around the world.

The International Fund for Animal Welfare (IFAW) was founded by a small group of people in
1969, to stop the commercial hunt for seal pups in Canada.

The International Tiger Day (also known as Global Tiger Day) is celebrated every year on 29
July to raise awareness for tiger conservation.

The IFAW recently organized major events on International Tiger conservation day.

You can read more about it at Q Source.

Q Source: http://www.ifaw.org/united-states/about-ifaw

72 The Food and Agriculture Organization (FAO) identifies which of these are traditional crops that “are
seemingly forgotten or are underutilized despite having outstanding nutritional or taste qualities”?

59
Total Marks : 200
Online Prelims TEST - 28 (TEXTBOOK)
( InsightsIAS Mock Test Series for UPSC Preliminary Exam 2020 ) Mark Scored : 0

1. Quinoa
2. Brown Rice
3. Common Buckwheat

Select the correct answer using the codes below.


A. 1 and 2 only
B. 2 and 3 only
C. 1 and 3 only
D. 1, 2 and 3

Correct Answer : C

Answer Justification :

Justification: All over the world local varieties of fruit, vegetables and grain are grown. Many are
seemingly forgotten or are underutilized despite having outstanding nutritional or taste qualities.
Some have good commercial potential and could be an excellent cash crop for a smallscale or family
farmer.

Amaranth, Moringa, Teff, Finger millet, Bambara groundnut, African garden eggplant, Cactus pear,
Common buckwheat, Yam bean, Breadfruit, Oca, Cardoon, Quinoa and Fe’i bananas are the
identified crops (by FAO).

For e.g. Amaranth is a native species to the Andean region of South America, including Argentina,
Peru and Bolivia. The leaves of the plant are frequently used in countries throughout Africa, the
Caribbean, India and China.

You can read about the rest at the Q Source.

Q Source: http://www.fao.org/traditional-crops/en/

73 Consider the following statements.


⌰〰〰
1. Industry 4.0 refers to trend towards automation and data exchange in manufacturing technologies
such as cloud computing and Internet of Things (IoT).
2. SAMARTH - Udyog Bharat 4.0 is an Industry 4.0 initiative of Department of Heavy Industry.

Select the correct answer using the codes below.


A. 1 only
B. 2 only
C. Both 1 and 2
D. None of the above

Correct Answer : C

Answer Justification :

60
Total Marks : 200
Online Prelims TEST - 28 (TEXTBOOK)
( InsightsIAS Mock Test Series for UPSC Preliminary Exam 2020 ) Mark Scored : 0

Justification: Statement 1: Although the terms "industry 4.0" and "fourth industrial revolution" are
often used interchangeably, "industry 4.0" factories have machines which are augmented with
wireless connectivity and sensors, connected to a system that can visualise the entire production
line and make decisions on its own.

In essence, industry 4.0 is the trend towards automation and data exchange in manufacturing
technologies and processes which include cyber-physical systems (CPS), the internet of things (IoT),
industrial internet of things (IIOT), cloud computing, cognitive computing and artificial intelligence.

The concept includes:

Smart manufacturing

Smart factory

Lights out (manufacturing) also known as dark factories

Industrial internet of things also called internet of things for manufacturing [7]

Statement 2: India has to move from manufacturing outfits of Industry 1.0 and 2.0 to Industry 4.0
and beyond. EEPC India in association of Department of Heavy industries (DHI) is raising
awareness on the 4th Industrial Revolution to drive the Indian manufacturing to a ‘Smart and
intelligent Manufacturing’ Hub said Mr Ravi Sehgal at a Industry 4.0 session .

Smart Advanced Manufacturing and Rapid Transformation Hub (SAMARTH) - Udyog Bharat 4.0 is
an Industry 4.0 initiative of Department of Heavy Industry, Government of India under its scheme
on Enhancement of Competitiveness in Indian Capital Goods Sector. The initiative aims to raise
awareness about Industry 4.0 among the Indian manufacturing industry through demonstration
centres. Currently there are four centres which include Center for Industry 4.0 (C4i4) Lab Pune;
IITD-AIA Foundation for Smart Manufacturing; I4.0 India at IISc Factory R & D Platform; Smart
Manufacturing Demo & Development Cell at CMTI.

Learning: Indian Engineering Exports have been growing at a rate of 10% with variation, but the
Engineering Exports as a percentage of ASEAN and World Exports is stagnating at 0.8-1 % over the
last 10-15 years. This is because majority of engineering goods originated from low or middle level
products.

Department of Commerce, Ministry of Commerce and Industry has given to this apex engineering
body a mandate to incarnate a Technology Centre to enable MSMEs to benefit from various new
technologies. India also needs to close a quality gap faced with the best in class, and leapfrogging to
newer technologies, will enable quality export products, hence EEPC India Technology Centre gains
ground.

Q Source:
https://economictimes.indiatimes.com/news/economy/policy/industry-4-0-making-india-smart-and-int

61
Total Marks : 200
Online Prelims TEST - 28 (TEXTBOOK)
( InsightsIAS Mock Test Series for UPSC Preliminary Exam 2020 ) Mark Scored : 0

elligent-manufacturing-hub/articleshow/70585241.cms

74 As per the Constitution of India, “Water, that is to say, water supplies" falls within the legislative
jurisdiction of the

A. Union government
B. State government
C. Concurrent list
D. Local government

Correct Answer : B

Answer Justification :

Justification: As per the Constitution of India, “Water, that is to say, water supplies" falls within
the legislative jurisdiction of the State Governments vide item 17 of the List II-State List under
Seventh Schedule referred to in the Article 246(3) of the Constitution and States are vested with the
constitutional right to plan, implement, operate and maintain water supply projects.

The Central Public Health and Environmental Engineering Organisation (CPHEEO) is Technical
Wing of the Ministry of Urban Development, Government of India, and deals with the matters
related to Urban Water Supply and Sanitation Including Solid Waste Management in the Country.

Though water supply and sanitation is a State subject, the policies, strategies and guidelines are
being provided by CPHEEO to the States & UTs Governments including Municipal Corporations /
Committees. The CPHEEO plays a vital role in processing the schemes posed for external funding
agencies including World Bank / JBIC/ ADB/ and Bilateral and Multilateral funding agencies and
institutional financing such as LIC.

It acts as an Advisory body at Central level to advise the concerned State agencies and Urban Local
Bodies (ULBs) in implementation, operation & maintenance of urban water supply, sanitation and
Solid Waste Management projects and helps to adopt latest technologies in these sub sectors.

Besides, the CPHEEO also implements centrally sponsored Accelerated Urban Water Supply
Programme (AUWSP) for small towns (scrutinizing / approving the schemes received from State
Departments from techno-economic angle), Solid Waste Management in 10 airfield towns, sponsors
research studies, organizes training courses for the in-service engineers working in the water
supply and sanitation sector.

Q Source: http://mohua.gov.in/cms/cpheeo.php

75 FAOLEX is an electronic collection of national laws, regulations and policies on


1. Food and agriculture
2. Natural resources management.

Select the correct answer using the codes below.

62
Total Marks : 200
Online Prelims TEST - 28 (TEXTBOOK)
( InsightsIAS Mock Test Series for UPSC Preliminary Exam 2020 ) Mark Scored : 0

A. 1 only
B. 2 only
C. Both 1 and 2
D. None of the above

Correct Answer : C

Answer Justification :

Justification: From 1952 to 1995, the FAO Legal Office published “Food and Agricultural
Legislation”, an annual compilation of significant, innovative and illustrative legislation. In 1995,
the Office overhauled this information service by replacing it with FAOLEX, becoming a forerunner
in the world of online legal databases.

FAOLEX is a database of national legislation, policies and bilateral agreements on food, agriculture
and natural resources management. It is constantly being updated, with an average of 8,000 new
entries per year. It currently contains legal and policy documents drawn from more than 200
countries, territories and regional economic integration organizations and originating in over 40
languages.

Administered by the Development Law Service (LEGN) of the FAO Legal Office, FAOLEX
complements FAO’s core function of advising its Members on legal and institutional means to
promote and regulate national development and international cooperation in the food and
agriculture sector.

Q Source: http://www.fao.org/faolex/background/en/

76 Consider the following statements.


1. The Agreement on Port State Measures (PSMA) is the first binding international agreement to
specifically target illegal, unreported and unregulated (IUU) fishing.
2. PMSA’s objective is to prevent and deter IUU fishing by preventing vessels engaged in IUU fishing
from using ports and landing their catches.

Select the correct answer using the codes below.


A. 1 only
B. 2 only
C. Both 1 and 2
D. None

Correct Answer : C

Answer Justification :

Justification: The Agreement on Port State Measures (PSMA) is the first binding international
agreement to specifically target illegal, unreported and unregulated (IUU) fishing.

63
Total Marks : 200
Online Prelims TEST - 28 (TEXTBOOK)
( InsightsIAS Mock Test Series for UPSC Preliminary Exam 2020 ) Mark Scored : 0

PSMA reduces the incentive of such vessels to continue to operate while it also blocks fishery
products derived from IUU fishing from reaching national and international markets.

The effective implementation of the PSMA ultimately contributes to the long-term conservation and
sustainable use of living marine resources and marine ecosystems. The provisions of the PSMA
apply to fishing vessels seeking entry into a designated port of a State which is different to their
flag State.

Learning: The PSMA entered into force in June 2016. By the first year and a half, more than a third
of countries were Parties to it.

The PSMA applies to fishing vessels seeking entry into a port other than those of their own State.

One in every five fish caught around the world every year is thought to originate from IUU fishing,
valued at $10 - $23 billion annually. Implementing the PSMA is one of the most cost-effective means
to curb IUU fishing.

Q Source: http://www.fao.org/port-state-measures/en/

77 Biosphere reserves are demarcated into 3 inter-related zones: Core, buffer and transition. Consider
the following statements.
1. The transition area is the outermost part of a biosphere reserve.
2. Transition area can accommodate settlements, crop lands, managed forests and area for intensive
recreation.
3. Only research activities are allowed in the buffer zone.
4. A core zone being National Park or Sanctuary is protected/regulated mostly under the Wildlife
(Protection) Act, 1972.

Select the correct answer using the codes below.


A. 1, 2, 3 and 4
B. 1, 2 and 4 only
C. 2, 3 and 4 only
D. 1 and 3 only

Correct Answer : B

Answer Justification :

Justification: Biosphere reserves are demarcated into following 3 inter-related zones:

Core Zone : Core zone must contain suitable habitat for numerous plant and animal species,
including higher order predators and may contain centres of endemism. Core areas often conserve
the wild relatives of economic species and also represent important genetic reservoirs having
exceptional scientific interest. A core zone being National Park or Sanctuary/protected/regulated
mostly under the Wildlife (Protection) Act, 1972. Whilst realizing that perturbation is an ingredient
of ecosystem functioning, the core zone is to be kept free from l human pressures external to the
system.

64
Total Marks : 200
Online Prelims TEST - 28 (TEXTBOOK)
( InsightsIAS Mock Test Series for UPSC Preliminary Exam 2020 ) Mark Scored : 0

Buffer Zone : The buffer zone, adjoins or surrounds core zone, uses and activities are managed in
this area in the ways that help in protection of core zone in its natural condition. These uses and
activities include restoration, demonstration sites for enhancing value addition to the resources,
limited recreation, tourism, fishing, grazing, etc; which are permitted to reduce its effect on core
zone. Research and educational activities are to be encouraged. Human activities, if natural within
BR, are likely to continue if these do not adversely affect the ecological diversity.

Transition Zone : The transition area is the outermost part of a biosphere reserve. This is usually
not delimited one and is a zone of cooperation where conservation knowledge and management
skills are applied and uses are managed in harmony with the purpose of the biosphere reserve. This
includes settlements, crop lands, managed forests and area for intensive recreation and other
economic uses characteristics of the region.

Q Source: https://vikaspedia.in/energy/environment/biodiversity-1/biosphere-reserves-in-india

78 Consider the following statements.


1. The Communal Award of 1932 was opposed by Indian National Congress.
2. The Award allotted minority communities a reserved number of seats in the legislatures to be
elected on the basis of a separate electorate.

Which of the above is/are correct?


A. 1 only
B. 2 only
C. Both 1 and 2
D. None

Correct Answer : C

Answer Justification :

Justification: The British policy of ‘Divide and Rule’ found another expression in the announcement
of the Communal Award in 1932.

The Award allotted to each minority a number of seats in the legislatures to be elected on the
basis of a separate electorate that is Muslims would be elected only by Muslims and Sikhs
only by Sikhs, and so on.

Muslims, Sikhs and Christians had already been treated as minorities. The Award declared
the Depressed Classes (Scheduled Castes of today) also to be a minority community entitled
to separate electorate and thus separated them from the rest of the Hindus.

The Congress was opposed to a separate electorate for Muslims, Sikhs and ‘Christians as it
encouraged the communal notion that they formed separate groups or communities having
interests different from the general body of Indians.

65
Total Marks : 200
Online Prelims TEST - 28 (TEXTBOOK)
( InsightsIAS Mock Test Series for UPSC Preliminary Exam 2020 ) Mark Scored : 0

Q Source: Revision: Chapter 23: India’s Struggle for Independence: Bipin Chandra

79 The polymer that is also known as “artificial silk” is

A. Vinyon
B. Modal
C. Rayon
D. Spandex

Correct Answer : C

Answer Justification :

Learning: In the present day, imitation silk may be made with rayon, mercerized cotton, polyester,
a blend of these materials, or a blend of rayon and silk.

Rayon is a manufactured fiber made from regenerated cellulose fiber. The many types and grades of
rayon can imitate the feel and texture of natural fibers such as silk, wool, cotton, and linen. The
types that resemble silk are often called artificial silk.

Rayon is made from purified cellulose, primarily from wood pulp.

Q Source: Science 8th NCERT

80 Consider the following statements.


1. President takes oath as the protector and defender of the Indian Constitution.
2. The oath is administered by Chief Justice of India or in his absence by the senior-most Judge of the
Supreme Court available.

Select the correct answer using the codes below.


A. 1 only
B. 2 only
C. Both 1 and 2
D. None of the above

Correct Answer : C

Answer Justification :

Justification: According to Article 60 of the Constitution of India, every President and every person
acting as President or discharging the functions of the President before entering upon office shall
make and subscribe oath to the office.

The oath inter alia includes the oath to preserve, protect and defend the constitution.

66
Total Marks : 200
Online Prelims TEST - 28 (TEXTBOOK)
( InsightsIAS Mock Test Series for UPSC Preliminary Exam 2020 ) Mark Scored : 0

In this sense, President takes oath as guardian of Indian Constitution.

The oath is administered by Chief Justice of India or in his absence by the senior-most Judge
of the Supreme Court available.

Q Source: IPML

81 Nations that border the Persian Gulf are


1. Saudi Arabia
2. United Arab Emirates
3. Iran
4. Kuwait
5. Jordan

Select the correct answer using the codes below.


A. 2 and 4 only
B. 1, 2, 3 and 4 only
C. 3 and 5 only
D. 1 and 5 only

Correct Answer : B

Answer Justification :

Justification:

Q Source: Map based questions: West Asia

67
Total Marks : 200
Online Prelims TEST - 28 (TEXTBOOK)
( InsightsIAS Mock Test Series for UPSC Preliminary Exam 2020 ) Mark Scored : 0

82 Private Entrepreneurs Guarantee (PEG) Scheme concerns

A. Manufacturing MSMEs
B. Warehousing capacity for food storage
C. Venture Capital Funds that have been established by the Government
D. Guaranteed Land acquisition for mega projects

Correct Answer : B

Answer Justification :

Learning: Under PEG, storage capacity is created by private parties, Central Warehousing
Corporation (CWC) and State Agencies for guaranteed hiring by FCI.

This is because the state alone cannot provide such facilities at all locations. It makes economic
sense for private players to finance the construction of these warehouses and then lease it out for
rent.

Q Source: www.dfpd.nic.in/peg.htm

83 It “imposes duty on [the] state to provide public assistance in cases of unemployment, old age,
sickness and disablement etc.”

A. Fundamental right
B. Directive Principle of State policy
C. Fundamental Duty
D. Convention

Correct Answer : B

Answer Justification :

Learning: The DPSPs are:

Article 38 says that the state will secure a social order for the promotion of welfare of the
people. Providing affordable healthcare is one of the ways to promote welfare.

Article 39(e) calls the state to make sure that health and strength of workers, men and
women, and the tender age of children are not abused.

Article 41 imposes duty on state to provide public assistance in cases of unemployment, old
age, sickness and disablement etc.

68
Total Marks : 200
Online Prelims TEST - 28 (TEXTBOOK)
( InsightsIAS Mock Test Series for UPSC Preliminary Exam 2020 ) Mark Scored : 0

Article 42 makes provision to protect the health of infant and mother by maternity benefit.

Article 47 make it duty of the state to improve public health, securing of justice, human
condition of works, extension of sickness, old age, disablement and maternity benefits and
also contemplated. Further, State’s duty includes prohibition of consumption of intoxicating
drinking and drugs are injurious to health.

Article 48A ensures that State shall Endeavour to protect and impose the pollution free
environment for good health.

Q Source: Page 29: Social and Political Life: 7th NCERT

84 The African network for the development of horticulture, "RADHORT", is an initiative of

A. FAO
B. International Food Policy Research Institute
C. IUCN
D. United Nations Environment Programme

Correct Answer : A

Answer Justification :

Justification: The African network for the development of horticulture "RADHORT", as an


independent intergovernmental entity is the result of a regional cooperation project for the
development of horticultural productions in Africa implemented since 1988 by FAO with funding of
the Kingdom of Belgium within the framework of the FAO-Belgium Cooperation program.

RADHORT reflects the will of the countries to promote the horticulture sector and to exchange
knowledge and mutual experiences. RADHORT member countries undertake to formulate a
strategic framework and an action plan to develop horticulture in support of the fight against
poverty and national commitments for food and nutritional security. Ten countries are behind
RADHORT and have signed the constitutive act,

Q Source: http://www.fao.org/in-action/radhort/fr/

85 With reference to the Housing and Urban Development Corporation Ltd. (HUDCO) and National
Buildings Construction Corporation Ltd (NBCCL), consider the following statements.
1. HUDCO was set up with the social motto of ‘Non-profit and Social Justice’.
2. HUDCO also addresses the rural housing shortage by lending for rural housing.
3. NBCCL is a blue-chip Government of India Navratna Enterprise under the Ministry of Housing and
Urban Affairs.

Select the correct answer using the codes below.

69
Total Marks : 200
Online Prelims TEST - 28 (TEXTBOOK)
( InsightsIAS Mock Test Series for UPSC Preliminary Exam 2020 ) Mark Scored : 0

A. 1 and 3 only
B. 2 and 3 only
C. 1 only
D. 3 only

Correct Answer : B

Answer Justification :

Justification: S1 and S2: The Housing and Urban Development Corporation Ltd. (HUDCO) was
established and incorporated on 25th April, 1970 as a fully owned enterprise of the Government of
India under the Companies Act 1956. It was set up with the social motto of ‘Profitability with Social
Justice’ and it has been serving the ‘un-served’ and thus bridging the ‘housing divide’ in the
country. HUDCO has been facilitating the housing & infrastructure development in the country with
a special thrust on meeting the housing needs of the ‘deprived’ i.e. Economically Weaker Sections
(EWS) and Low-Income Groups (LIG). HUDCO also addresses the rural housing shortage by lending
for rural housing.

Apart from the financing operations, HUDCO offers design and consultancy services in Consultancy
in the field of Architecture, Urban Design, Landscape, Urban and Regional Planning, Transportation
Planning and Infrastructure development.

HUDCO plays a key role in various flagship programmes of the government to develop the Indian
housing and urban infrastructure sectors, such as PMAY-Housing for All (Urban) under which
HUDCO has been appointed as an appraisal agency for the three components, i.e. in-situ slum
redevelopment (using land as a resource), Affordable Housing in Partnership (AHP) and Beneficiary
Led Construction (BLC) projects. HUDCO has also been designated as one of the Central Nodal
Agencies (CNA) under CLSS, which is a component of the Pradhan Mantri Awas Yojana (PMAY-
Urban) to channelize the subsidy.

Under the disinvestment programme of Government of India, 10.193% of equity shareholding of


Government of India has been disinvested through Initial Public Offer (IPO).

S3: NBCC (India) Limited, formerly known as National Buildings Construction Corporation Ltd., is a
blue-chip Government of India Navratna Enterprise under the Ministry of Housing and Urban
Affairs. Listed with both the Stock Exchanges, the company’s unique business model has today,
made it stand out as a leader in its own right in the construction sector with more than INR 36000
Crore Order Book in hand till March 2016.

Learning: The National Buildings Organisation (NBO) was established in 1954 as an attached office
under the Ministry of Housing and Urban Affairs(the then Ministry of Works and Housing) for
technology transfer, experimentation, development and dissemination of housing statistics. In the
context of formulation of Housing Policy and Programmes, study of socio-economic aspects of
housing issues and the increased need for housing statistics, NBO was restructured in 1992. NBO
was further restructured in March, 2006 with the revised mandate keeping in view the current
requirements under the National Housing Policy, and various socio-economic and statistical
functions connected with housing and building activities.

70
Total Marks : 200
Online Prelims TEST - 28 (TEXTBOOK)
( InsightsIAS Mock Test Series for UPSC Preliminary Exam 2020 ) Mark Scored : 0

Q Source: http://mohua.gov.in/cms/psus.php

http://mohua.gov.in/cms/NBO.php

86 In what type of soil is Magnesium deficiency most likely to occur due to leaching?

A. Clayey soil
B. Light and sandy soils
C. Red soil
D. Equally likely in all the above

Correct Answer : B

Answer Justification :

Justification: In strongly acidic, light, sandy soils magnesium can be easily leached away.

In all other soils, where adhesion, denseness or less leaching occurs, magnesium may not be
leached.

Magnesium is an essential macro nutrient found from 0.2-0.4% dry matter and is necessary
for normal plant growth.

Magnesium has an important role in photosynthesis because it forms the central atom of
chlorophyll. Therefore, without sufficient amounts of magnesium, plants begin to degrade the
chlorophyll in the old leaves

Low amounts of Mg lead to a decrease in photosynthetic and enzymatic activity within the
plants.

Plants deficient in magnesium also produce smaller, woodier fruits.

Excess potassium, generally due to fertilizers, further aggravates the stress from the
magnesium deficiency, as does aluminium toxicity.

Applying home-made compost mulch can prevent leaching during excessive rainfall and
provide plants with sufficient amounts of nutrients, including magnesium

Q Source: AR: Page 52: Science 8th NCERT

87 Eleven of the India’s eighteen biosphere reserves are a part of the World Network of Biosphere Reserves, based on the UNESCO Man
and the Biosphere (MAB) Programme list. The latest addition is

71
Total Marks : 200
Online Prelims TEST - 28 (TEXTBOOK)
( InsightsIAS Mock Test Series for UPSC Preliminary Exam 2020 ) Mark Scored : 0

A. Agasthyamalai Biosphere Reserve


B. Achanakmar-Amarkantak Biosphere Reserve
C. Great Nicobar Biosphere Reserve
D. Khangchendzonga National Park

Correct Answer : D

Answer Justification :

Justification: Khangchendzonga National Park also Kanchenjunga Biosphere Reserve is a National


Park and a Biosphere reserve located in Sikkim, India. It was inscribed to the UNESCO World
Heritage Sites list in July 2016, becoming the first "Mixed Heritage" site of India. It was included in
the UNESCO Man and the Biosphere Programme in 2018.

The park gets its name from the mountain Kangchenjunga (alternative spelling Khangchendzonga)
which is 8,586 metres tall, the third-highest peak in the world. The total area of this park is 849.5
km2.

The park contains many mammal species including musk deer, snow leopard, Himalayan tahr, wild
dog, sloth bear, civet, Himalayan black bear, red panda, Tibetan wild ass, Himalayan blue sheep,
serow, goral and takin, as well as reptiles including rat snake and Russell's viper.

Q Source: https://en.wikipedia.org/wiki/Khangchendzonga_National_Park

88 The "Three Sisters" recognized by the World Trade Organization's (WTO) Sanitary and Phytosanitary
Measures (SPS) Agreement include
1. International Plant Protection Convention (IPPC)
2. Codex Alimentarius Commission standards for food
3. World Organization for Animal Health (OIE) standards for animal health

Select the correct answer using the codes below.


A. 1 only
B. 2 only
C. 1 and 3 only
D. 1, 2 and 3

Correct Answer : D

Answer Justification :

Justification: The International Plant Protection Convention (IPPC) is an intergovernmental treaty


signed by over 180 countries, aiming to protecting the world's plant resources from the spread and
introduction of pests, and promoting safe trade. The Convention introduced International Standards
for Phytosanitary Measures (ISPMs) as its main tool to achieve its goals, making it the sole global
standard setting organization for plant health.

72
Total Marks : 200
Online Prelims TEST - 28 (TEXTBOOK)
( InsightsIAS Mock Test Series for UPSC Preliminary Exam 2020 ) Mark Scored : 0

The IPPC is one of the "Three Sisters" recognized by the World Trade Organization's (WTO)
Sanitary and Phytosanitary Measures (SPS) Agreement, along with the Codex Alimentarius
Commission for food safety standards and the World Organization for Animal Health (OIE) for
animal health standards.

Learning: The Commission on Phytosanitary Measures (CPM) was established in 2005 building on
the Committee of Experts on Phytosanitary Measures (CEPM), whose first session was held in 1993.
The CPM is the IPPC main governing body, holding its sessions annually.

The CPM promotes the full implementation of the Convention’s objectives, including reviewing the
state of plant protection in the world and actions to control the international spread of pests and
their introduction into endangered areas; establishing and keeping under review the necessary
institutional arrangements and procedures for the development and adoption of international
standards; and adopting international standards.

Q Source: https://www.ippc.int/en/about/overview/

89 Coal Gas contains a variety of gases including


1. Hydrogen
2. Methane
3. Nitrogen
4. Carbon dioxide

Select the correct answer using the codes below.


A. 2, 3 and 4 only
B. 1 and 4 only
C. 1, 2, 3 and 4
D. 1, 2 and 3 only

Correct Answer : C

Answer Justification :

Justification: Coal gas contains a variety of calorific gases including hydrogen, carbon monoxide,
methane and volatile hydrocarbons together with small quantities of non-calorific gases such as
carbon dioxide and nitrogen.

Coal gas is a flammable gaseous fuel made from coal and supplied to the user via a piped
distribution system.

The gas obtained when coal is heated strongly in the absence of air is called coal gas.

Coal gasification is the process of producing syngas–a mixture consisting primarily of carbon
monoxide (CO), hydrogen (H2), carbon dioxide (CO2), methane (CH4), and water vapour
(H2O)–from coal and water, air and/or oxygen.

Q Source: AR: Page 57: Science 8th NCERT

73
Total Marks : 200
Online Prelims TEST - 28 (TEXTBOOK)
( InsightsIAS Mock Test Series for UPSC Preliminary Exam 2020 ) Mark Scored : 0

90 Consider the following statements.


1. Atlantic region dominates the aquaculture sector production-wise owing to its indented coastlines.
2. The Blue Growth Initiative is World Bank’s framework for sustainably developing fisheries and
aquaculture.
3. The concept of “Blue Growth” came out of Rio + 20 negotiations.

Select the correct answer using the codes below.


A. 2 only
B. 3 only
C. 1, 2 and 3
D. None of the above

Correct Answer : D

Answer Justification :

Justification: Statement 1: In 2017 the Asia–Pacific region continued to dominate the aquaculture
sector, accounting for 91.9 percent of global production.

Some facts:

In the 1970s, aquaculture produced about 3 million tonnes of fish

By 2017, world aquaculture production reached 80.1 million tonnes valued at USD 237.5 billion
(farm-gate value)

It employs some 26 million workers, 19.3 million directly and about 6.5 million indirectly

Statement 2: The Blue Growth Initiative is FAO’s framework for sustainably developing fisheries
and aquaculture. Blue Growth differs from business as usual in the fisheries and aquaculture
sectors, which historically focused on single interests, such as producing more fish or safeguarding
the environment, and did not prioritize social benefits.

Statement 3: S3 is incorrect. The concept of Blue Growth is similar (not the same) in many respects
to that of the Blue Economy—a concept that came out of Rio +20—in that both center on the pillars
of sustainable development: environmental, economic, and social. FAO uses the term Blue Growth
to emphasize the need for growth in many Member States particularly in the fisheries and
aquaculture sectors.

The goals of the Blue Growth Initiative are to maximize economic and social benefits while
minimizing environmental degradation from these sectors. These goals are closely aligned with the
2030 Agenda for Sustainable Development (supported by the Sustainable Development
Goals—SDGs).

Q Source: http://www.fao.org/aquaculture/en/

http://www.fao.org/3/a-i7862e.pdf

74
Total Marks : 200
Online Prelims TEST - 28 (TEXTBOOK)
( InsightsIAS Mock Test Series for UPSC Preliminary Exam 2020 ) Mark Scored : 0

91 Which of these are biodegradable materials?


1. Tissue paper
2. Cotton cloth
3. Glass objects
4. Ball-point pen refills

Select the correct answer using the codes below.


A. 1 and 2 only
B. 2 and 4 only
C. 1, 2 and 3 only
D. 1, 3 and 4 only

Correct Answer : A

Answer Justification :

Justification: The waste materials which cannot be broken down into non-poisonous or harmless
substances in nature are called non-biodegradable waste. Examples are plastics, polythene bags,
ball-point pen refills, synthetic fibres, and glass objects, metal articles like aluminium cans, iron
nails, silver foil and radioactive wastes.

Cotton cloth, paper, woollen clothes, wood etc. are bio-degradable. Some of the common examples
of bio-degradable material are as follows: Paper bags; cardboard Boxes; Paper cups and plates; Jute
products; Cotton products; Tissue paper; Plates and trays made up of Leaves.

Q Source: Page 38: Science 8th NCERT

92 With reference to COVID-19 and related facts, consider the following statements.
1. Viruses are named based on their genetic structure to facilitate the development of diagnostic tests,
vaccines and medicines.
2. COVID-19 is also officially known as severe acute respiratory syndrome coronavirus 2 (SARS-
CoV-2).
3. Middle East Respiratory Syndrome (MERS) is caused by coronaviruses.
4. As of February, 2020, there is no vaccine and no specific antiviral medicine to prevent or treat
COVID-2019.

Select the correct answer using the codes below.


A. 1 and 2 only
B. 1, 2 and 4 only
C. 1, 2, 3 and 4
D. 3 and 4 only

Correct Answer : C

Answer Justification :

75
Total Marks : 200
Online Prelims TEST - 28 (TEXTBOOK)
( InsightsIAS Mock Test Series for UPSC Preliminary Exam 2020 ) Mark Scored : 0

Justification: Statement 1 and 2: Viruses, and the diseases they cause, often have different names.
For example, HIV is the virus that causes AIDS. People often know the name of a disease, such as
measles, but not the name of the virus that causes it (rubeola).

There are different processes, and purposes, for naming viruses and diseases.

Viruses are named based on their genetic structure to facilitate the development of diagnostic tests,
vaccines and medicines. Virologists and the wider scientific community do this work, so viruses are
named by the International Committee on Taxonomy of Viruses (ICTV). COVID-19 is also known as
SARS-Cov-2.

Statement 3: Coronaviruses are a large family of viruses which may cause illness in animals or
humans. In humans, several coronaviruses are known to cause respiratory infections ranging from
the common cold to more severe diseases such as Middle East Respiratory Syndrome (MERS) and
Severe Acute Respiratory Syndrome (SARS). The most recently discovered coronavirus causes
coronavirus disease COVID-19.

The virus that causes COVID-19 and the one that caused the outbreak of Severe Acute Respiratory
Syndrome (SARS) in 2003 are related to each other genetically, but the diseases they cause are
quite different.

SARS was more deadly but much less infectious than COVID-19. There have been no outbreaks of
SARS anywhere in the world since 2003.

Statement 4: To date, there is no vaccine and no specific antiviral medicine to prevent or treat
COVID-2019. However, those affected should receive care to relieve symptoms. People with serious
illness should be hospitalized. Most patients recover thanks to supportive care.

Possible vaccines and some specific drug treatments are under investigation. They are being tested
through clinical trials. WHO is coordinating efforts to develop vaccines and medicines to prevent
and treat COVID-19.

Q Source:
https://www.who.int/emergencies/diseases/novel-coronavirus-2019/technical-guidance/naming-the-c
oronavirus-disease-(covid-2019)-and-the-virus-that-causes-it

https://www.who.int/news-room/q-a-detail/q-a-coronaviruses

93 The proposal of the Cripps mission included


1. After the end of the First World War, a constituent assembly would be convened to frame a new
constitution.
2. Any province not willing to join the Indian Union could have a separate constitution and form a
separate Union.

Which of the above is/are correct?


A. 1 only
B. 2 only
C. Both 1 and 2
D. None
76
Total Marks : 200
Online Prelims TEST - 28 (TEXTBOOK)
( InsightsIAS Mock Test Series for UPSC Preliminary Exam 2020 ) Mark Scored : 0

Correct Answer : B

Answer Justification :

Learning: The main proposals of the mission were as follows:

An Indian Union with a dominion status; would be set up; it would be free to decide its
relations with the Commonwealth and free to participate in the United Nations and other
international bodies.

After the end of the Second World war (not first), a constituent assembly would be convened
to frame a new constitution. Members of this assembly would be partly elected by the
provincial assemblies through proportional representation and partly nominated by the
princes.

The British Government would accept the new constitution subject to two conditions.

any province not willing to join the Union could have a separate constitution and form a
separate Union, and (ii) the new constitution- making body and the British Government
would negotiate a treaty to effect the transfer of power and to safeguard racial and
religious minorities.

In the meantime, defence of India would remain in British hands and the governor-general’s
powers would remain intact.

Q Source: Chapter 35: India’s Struggle for Independence: Bipin Chandra

94 Which of the following is/are the main functions of the Review Committee on Genetic Manipulation
(RCGM)?
1. To review all on going r-DNA projects involving high risk category and controlled field experiments
2. To lay down guidelines for producers with regards to production, sale, import and use of GMOs both
for research and applications

Select the correct answer using the codes below.


A. 1 only
B. 2 only
C. Both 1 and 2
D. None of the above

Correct Answer : C

77
Total Marks : 200
Online Prelims TEST - 28 (TEXTBOOK)
( InsightsIAS Mock Test Series for UPSC Preliminary Exam 2020 ) Mark Scored : 0

Answer Justification :

Justification: Main functions of RGCM are:

To bring out manuals of guidelines specifying producers for regulatory process on GMOs in
research, use and applications including in industry with a view to ensure environmental
safety.

To review all on going r-DNA projects involving high risk category and controlled field
experiments

To lay down producers for restriction or prohibition, production, sale, import & use of GMOs
both for research and applications.

To authorize imports of GMOs/ transgenes/ transgenic seeds for research purposes.

Learning: Institutional Biosafety Committees (IBSCs) main function is to note, examine and
approve proposals involving r-DNA work; to ensure adherence of r-DNA Safety Guidelines- 1990 of
Government and inspection of containment facilities at R&D and production units.

GEAC is the apex body constituted in the Ministry of Environment and Forests.

It was set up under 'Rules for Manufacture, Use, Import, Export and Storage of Hazardous
Microorganisms/Genetically Engineered Organisms or Cells 1989', under the Environment
Protection Act, 1986.

The Rules of 1989 also define five competent authorities i.e. IBSC, RCGM, GEAC, State
Biotechnology Coordination Committee (SBCC) and District Level Committee (DLC) for
handling of various aspects of the rules.

Q Source: MoEFCC Website

95 In India, majority of growing stock of Bamboo is in the

A. Southern India
B. Western India
C. Central India
D. North-eastern region

Correct Answer : D

78
Total Marks : 200
Online Prelims TEST - 28 (TEXTBOOK)
( InsightsIAS Mock Test Series for UPSC Preliminary Exam 2020 ) Mark Scored : 0

Answer Justification :

Learning: The table below shows the regional stock of Bamboo in India.

28% of area and 66% of growing stock of bamboo in NE region. 20% of area and 12% of growing
stock in MP & Chattisgarh.

It is widely seen as a wood substitute in India given that we import a lot of wood. Largest use is in
scaffolding, second largest in paper, third largest handicrafts (see distribution in image below)

Also see http://nbm.nic.in/grow_bamboo.html

Q Source: Based on past year UPSC papers

96 The general policy of the Indian National Congress (INC) towards the Indian princely states was first
enunciated in

A. 1930 at Lahore Congress


B. 1920 at Nagpur Congress
C. 1919 at Lucknow Session
D. 1905 at Kolkata Session

Correct Answer : B

Answer Justification :

Justification: The general policy of the Indian National Congress (INC) towards the Indian princely
states was first enunciated in 1920 at Nagpur and later carried forward till independence. It was
first of exhortation for a responsible government in the states and later active political intervention.

It called upon the Princes to grant full responsible government in their States. However, no direct
political activity was to be resorted for.

79
Total Marks : 200
Online Prelims TEST - 28 (TEXTBOOK)
( InsightsIAS Mock Test Series for UPSC Preliminary Exam 2020 ) Mark Scored : 0

This position continued till 1935.

Two developments in the mid-30s brought a radical change in the relations between the
Princely States and British India. The Government of India Act of 1935 projected a scheme of
federation in which the States were brought into a direct constitutional relationship with
British India.

The second major impact on the States was created by the acceptance of office by the
Congress in majority of the British Indian provinces in 1937.

The installation of the Congress ministries in the neighbouring British Indian provinces
encouraged the Praja Mandal leaders to step up their political activities for demanding
responsible government in the Princely States.

Q Source: Revision: Chapter 28: India’s Struggle for Independence: Bipin Chandra

97 With respect to International Energy Agency (IEA), consider the following statements.
1. IEA is an inter-governmental organization established as per framework of the Organisation for
Economic Co-operation and Development (OECD).
2. It was established in the wake of the 1973 oil crisis.
3. India is a member of IEA.

Select the correct answer using the codes below.


A. 1 and 2 only
B. 2 and 3 only
C. 1 only
D. 1, 2 and 3

Correct Answer : A

Answer Justification :

Justification: Statement 1: IEA is an inter-governmental organization established in 1974 as per


framework of the Organisation for Economic Co-operation and Development (OECD).

It was established in the wake of the 1973 oil crisis after the OPEC cartel had shocked the world
with a steep increase in oil prices.

The IEA was established as the main international forum for energy co-operation on a variety of
issues such as security of supply, long-term policy, information transparency, energy efficiency,
sustainability, research and development, technology collaboration, and international energy
relations.

Statement 3: A candidate country to the IEA must be a member country of the OECD; India is not.

80
Total Marks : 200
Online Prelims TEST - 28 (TEXTBOOK)
( InsightsIAS Mock Test Series for UPSC Preliminary Exam 2020 ) Mark Scored : 0

The IEA is made up of 30 member countries. In addition, thanks to its successful open door policy to
emerging countries, the IEA family also includes eight association countries. Two countries are
seeking accession to full membership, Chile and Lithuania.

Q Source: https://www.iea.org/about/history

98 Consider the following about Multi Commodity Exchange (MCX) of India.


1. It operates outside of the regulatory purview of SEBI.
2. It is India’s first exchange to offer commodity options contracts.

Select the correct answer using the codes below.


A. 1 only
B. 2 only
C. Both 1 and 2
D. None

Correct Answer : B

Answer Justification :

The Multi Commodity Exchange of India Limited (MCX),- India’s first listed exchange, is a state-of-
the-art, commodity derivatives exchange that facilitates online trading of commodity derivatives
transactions, thereby providing a platform for price discovery and risk management. The Exchange,
which started operations in November 2003, operates under the regulatory framework of Securities
and Exchange Board of India (SEBI).

MCX offers trading in commodity derivative contracts across varied segments including bullion,
industrial metals, energy and agricultural commodities. It is India’s first exchange to offer
commodity options contracts. The Exchange focuses on providing commodity value chain
participants with neutral, secure and transparent trade mechanisms, and formulates quality
parameters and trade regulations, in conformity with the regulatory framework.

The Exchange’s flagship index series, MCX iCOMDEX, is a series of real-time commodity futures
price indices, which give information on market movements in key commodities traded on MCX.

Globally, MCX ranks no. 1 in silver, no. 2 in natural gas, no. 3 in crude oil and gold in futures
trading.

Q Source: Frequently in news

99 Peatlands are critical for the environment because


1. They are valuable carbon stores.
2. They host rich and complex biodiversity.

Select the correct answer using the codes below.


A. 1 only

81
Total Marks : 200
Online Prelims TEST - 28 (TEXTBOOK)
( InsightsIAS Mock Test Series for UPSC Preliminary Exam 2020 ) Mark Scored : 0

B. 2 only
C. Both 1 and 2
D. None

Correct Answer : C

Answer Justification :

Justification: The complex biodiversity of peatlands means they are home to variety of species.
Their high carbon content makes them uniquely vulnerable to incineration if they are drained.

They are globally important carbon store. The unregulated exploitation of peatlands can potentially
be detrimental to environment and to climate, as it could release carbon emissions that have been
locked in for millennia.

Learning and Context: The Brazzaville declaration was signed to promote better management and
conservation world’s largest tropical peatlands-Cuvette Centrale region in Congo Basin from
unregulated land use and prevent its drainage and degradation.

It was signed jointly by Democratic Republic of Congo (DRC), Republic of Congo and Indonesia on
the sidelines of Third Partners Meeting of Global Peatlands Initiative held in Brazzaville, Republic of
Congo.

The Cuvette Centrale region in Congo Basin is world’s largest natural tropical peatlands, which are
about size of England. It stores three years equivalent of global greenhouse gas emissions.

Q Source: Based on past year UPSC papers

100 Who can authorise the use of Hindi or any other official language of the state, in the proceedings in
the high court of the state?

A. Chief Justice of India


B. Chief Justice of the concerned High Court
C. Governor of the concerned state, with the previous consent of the President
D. It cannot be authorized.

Correct Answer : C

Answer Justification :

Justification: The constitutional provisions dealing with the language of the courts and legislation
are as follows:

(i) Until Parliament provides otherwise, the following are to be in the English language only:

All proceedings in the Supreme Court and in every high court.

82
Total Marks : 200
Online Prelims TEST - 28 (TEXTBOOK)
( InsightsIAS Mock Test Series for UPSC Preliminary Exam 2020 ) Mark Scored : 0

The authoritative texts of all bills, acts, ordinances, orders, rules, regulations and byelaws at
the Central and state levels.

However, the governor of a state, with the previous consent of the president, can authorise
the use of Hindi or any other official language of the state, in the proceedings in the high court of
the state, but not with respect to the judgements, decrees and orders passed by it.

Learning: The Official Language Act of 1963 lays down that Hindi translation of acts, ordinances,
orders, regulations and bye-laws published under the authority of the president are deemed to be
authoritative texts. Further, every bill introduced in the Parliament is to be accompanied by a Hindi
translation. Similarly, there is to be a Hindi translation of state acts or ordinances in certain cases.

Q Source: Ch 58: IPML

83

You might also like